Maternity Exam Ch. 6,8, 27-31

¡Supera tus tareas y exámenes ahora con Quizwiz!

Which substance used during pregnancy causes vasoconstriction and decreased placental perfusion, resulting in maternal and neonatal complications? a. Alcohol b. Caffeine c. Tobacco d. Chocolate

c. Tobacco Smoking in pregnancy is known to cause a decrease in placental perfusion and is the cause of low-birth-weight infants. Prenatal alcohol exposure is the single greatest preventable cause of mental retardation. Alcohol use during pregnancy can cause high blood pressure, miscarriage, premature birth, stillbirth, and anemia. Caffeine may interfere with certain medications and worsen arrhythmias. Chocolate, particularly dark chocolate, contains caffeine that may interfere with certain medications.

Since the gene for cystic fibrosis was identified in 1989, data can be collected for the purposes of genetic counseling for couples regarding carrier status. According to the most recent statistics, how often does cystic fibrosis occur in Caucasian live births? a. 1 in 100 b. 1 in 1000 c. 1 in 2000 d. 1 in 3200

d. 1 in 3200 Cystic fibrosis occurs in approximately 1 in 3200 Caucasian live births. 1 in 100, 1 in 1000, and 1 in 2000 occurrences of cystic fibrosis in live births are all too frequent rates.

What is the most dangerous effect on the fetus of a mother who smokes cigarettes while pregnant? a. Genetic changes and anomalies b. Extensive CNS damage c. Fetal addiction to the substance inhaled d. Intrauterine growth restriction

d. Intrauterine growth restriction The major consequences of smoking tobacco during pregnancy are low-birth-weight infants, prematurity, and increased perinatal loss. Cigarettes will not normally cause genetic changes or extensive CNS damage. Addiction to tobacco is not a usual concern related to the neonate.

Which statement regarding the term contraceptive failure rate is the most accurate? a. The contraceptive failure rate refers to the percentage of users expected to have an accidental pregnancy over a 5-year span. b. It refers to the minimum rate that must be achieved to receive a government license. c. The contraceptive failure rate increases over time as couples become more careless. d. It varies from couple to couple, depending on the method and the users.

d. It varies from couple to couple, depending on the method and the users. Contraceptive effectiveness varies from couple to couple, depending on how well a contraceptive method is used and how well it suits the couple. The contraceptive failure rate measures the likelihood of accidental pregnancy in the first year only. Failure rates decline over time because users gain experience.

Which alteration in cyclic bleeding best describes bleeding that occurs at any time other than menses? a. Oligomenorrhea b. Menorrhagia c. Leiomyoma d. Metrorrhagia

d. Metrorrhagia Metrorrhagia (intermenstrual bleeding) refers to any episode or degree of bleeding that occurs between periods. It may be caused by contraceptives that contain progesterone or by intrauterine devices (IUDs). Oligomenorrhea is infrequent or scanty menstruation. Menorrhagia is excessive menstruation. Leiomyoma is a common cause of excessive bleeding.

What form of heart disease in women of childbearing years generally has a benign effect on pregnancy? a. Cardiomyopathy b. Rheumatic heart disease c. Congenital heart disease d. Mitral valve prolapse

d. Mitral valve prolapse Mitral valve prolapse is a benign condition that is usually asymptomatic. Cardiomyopathy produces congestive heart failure during pregnancy. Rheumatic heart disease can lead to heart failure during pregnancy. Some congenital heart diseases produce pulmonary hypertension or endocarditis during pregnancy.

Which congenital anomalies can occur because of the use of antiepileptic drugs (AEDs) in pregnancy? (Select all that apply.) a. Cleft lip b. Congenital heart disease c. Neural tube defects d. Gastroschisis e. Diaphragmatic hernia

A, B, C Congenital anomalies that can occur with AEDs include cleft lip or palate, congenital heart disease, urogenital defects, and neural tube defects. Carbamazepine and valproate should be avoided if all possible; they may cause neural tube defects. Congenital anomalies of gastroschisis and diaphragmatic hernia are not associated with the use of AEDs.

The nurse, responsible for providing health teaching regarding the side effects of combined oral contraceptives (COCs), should discuss what issues? (Select all that apply.) a. Gallbladder disease b. Myocardial infarction c. Hypotension d. Breast tenderness e. Dry skin and scalp

A, B, D Serious side effects include stroke, myocardial infarction, hypertension, gallbladder disease, and liver tumors. More common side effects include nausea, breast tenderness, fluid retention, increased appetite, and oily skin and scalp

The nurse suspects that a client has early signs of ectopic pregnancy. The nurse should be observing the client for which signs or symptoms? (Select all that apply.) a. Pelvic pain b. Abdominal pain c. Unanticipated heavy bleeding d. Vaginal spotting or light bleeding e. Missed period

A, B, D, E A missed period or spotting can be easily mistaken by the client as an early sign of pregnancy. More subtle signs depend on exactly where the implantation occurs. The nurse must be thorough in her assessment because pain is not a normal symptom of early pregnancy. As the fallopian tube tears open and the embryo is expelled, the client often exhibits severe pain accompanied by intraabdominal hemorrhage, which may progress to hypovolemic shock with minimal or even no external bleeding. In approximately one half of women, shoulder and neck pain results from irritation of the diaphragm from the hemorrhage.

Autoimmune disorders often occur during pregnancy because a large percentage of women with an autoimmune disorder are of childbearing age. Which disorders fall into the category of collagen vascular disease? (Select all that apply.) a. Multiple sclerosis b. Systemic lupus erythematosus (SLE) c. Antiphospholipid syndrome d. Rheumatoid arthritis e. Myasthenia gravis

B, C, D, E Multiple sclerosis is not an autoimmune disorder. This patchy demyelination of the spinal cord may be a viral disorder. Autoimmune disorders (collagen vascular disease) make up a large group of conditions that disrupt the function of the immune system of the body. These disorders include those listed, as well as systemic sclerosis.

Reports have linked third trimester use of selective serotonin uptake inhibitors (SSRIs) with a constellation of neonatal signs. The nurse is about to perform an assessment on the infant of a mother with a history of a mood disorder. Which signs and symptoms in the neonate may be the result of maternal SSRI use? (Select all that apply.) a. Hypotonia b.Hyperglycemia c. Shivering d. Fever e. Irritability

C, D, E Neonatal signs of maternal SSRI use include continuous crying, irritability, jitteriness, shivering, fever, hypertonia, respiratory distress, feeding difficulty, hypoglycemia, and seizures. The onset of signs and symptoms ranges from several hours to several days after birth, but the signs generally resolve within 2 weeks.

When a nurse is counseling a woman experiencing primary dysmenorrhea, which nonpharmacologic intervention might be recommended? a. Increasing the intake of red meat to replace blood loss b. Reducing the intake of diuretic foods, such as peaches and asparagus c. Temporarily substituting physical activity for a sedentary lifestyle d. Using a heating pad on the abdomen to relieve cramping

d. Using a heating pad on the abdomen to relieve cramping Heat minimizes cramping by increasing vasodilation and muscle relaxation and minimizing uterine ischemia. Dietary changes such as a low-fat vegetarian diet may be recommended for women experiencing dysmenorrhea. Increasing the intake of diuretics, including natural diuretics such as asparagus, cranberry juice, peaches, parsley, and watermelon, may help ease the symptoms associated with dysmenorrhea. Exercise has been found to help relieve menstrual discomfort through increased vasodilation and subsequent decreased ischemia.

MATCHING Because most pregnant women continue their usual activities, trauma remains a common complication during pregnancy. Approximately 1 in 12 pregnancies in the United States are complicated by trauma each year. Because of the physiologic alterations that accompany pregnancy, special considerations for the mother and fetus are necessary when trauma occurs. Match the maternal system adaptation in pregnancy with the clinical response to trauma. a. Increased oxygen consumption b. Increased heart rate c. Decreased gastric motility d. Displacement of abdominal viscera e. Increase in clotting factors 1. Decreased placental perfusion in the supine position 2. Increased risk of thrombus formation 3. Altered pain referral 4. Increased risk of acidosis 5. Increased risk of aspiration

1. B 2. E 3. D 4. A 5. C

Matching: Herbal preparations have long been used for the management of menstrual problems, including dysmenorrhea, cramping and discomfort, and breast pain. For the nurse to counsel adequately the client who elects to use this alternative modality, understanding the action of these herbal preparations is important. Match the herbal medicine with the appropriate action. a. Uterine antispasmodic b. Uterotonic c. Antiinflammatory d. Estrogen-like luteinizing hormone suppressant e. Decreases prolactin levels 1. Fennel, dong quai 2. Chaste tree fruit 3. Black cohosh root 4. Valerian, wild yam 5. Ginger

1. B 2. E 3. D 4. A 5. C

MATCHING The nurse is preparing to teach an antepartum client with GDM the correct method of administering an intermediate-acting insulin, such as neutral protamine Hagedorn (NPH), with a short-acting insulin (regular). In the correct order from 1 through 6, match the step number with the action needed to teach the client self-administration of this combination of insulin. a. Without adding air, withdraw the correct dose of NPH insulin. b. Gently rotate the insulin to mix it and wipe the stopper. c. Inject air equal to the dose of NPH insulin into the vial and remove the syringe. d. Inject air equal to the dose of regular insulin into the vial and withdraw the medication. e. Check the insulin bottles for the expiration date. f. Wash hands. 1. Step 1 2. Step 2 3. Step 3 4. Step 4 5. Step 5 6. Step 6

1. F 2. E 3. B 4. C 5. D 6. A

Which comment by the nurse would be most helpful in instruct the woman on how best to reduce her risk of developing toxic shock syndrome (TSS)? (Select all that apply.) a. "You should always leave your diaphragm in place for at 6 hours after intercourse." b. "You should remove your diaphragm right after intercourse to prevent TSS." c. "You can use your diaphragm during your menstrual cycle as long as you clean it well." d. "Report a fever of 38.4 0 C to your health care provider immediately." e. "Make sure you don't leave your diaphragm in for longer than 24 hours"

A The nurse should instruct the client on the proper use and removal of the diaphragm and include the danger signs of TSS. The diaphragm must remain against the cervix for 6 to 8 hours to prevent pregnancy, but it should not remain in place longer than 8 hours to avoid the risk of TSS. The diaphragm should not be used during menses. A fever of 38.4 0 C or higher could be a symptom of TSS and should be reported immediately.

Which system responses would the nurse recognize as being related to prostaglandin (PGF2) release? (Select all that apply) a. Systemic responses b. Gastrointestinal system c. Central nervous system d. Respiratory System e. Genitourinary system

A, B, C Systemic responses to PGF2 include backache, weakness, and sweating. Gastrointestinal system changes include nausea, vomiting, anorexia, and diarrhea. Central nervous system changes manifest themselves as dizziness, syncope, headache, and poor concentration; they usually begin at the onset of menstruation and last 8 to 48 hours. PGF2 as no reportable effects on the genitourinary or respiratory systems.

While educating the client regarding the risks and benefits of a vasectomy, which information should the nurse include? (Select all that apply.) a. Sterilization should be performed under general anesthesia. b. Pain, bleeding, and infection are possible complications. c. For several months, pregnancy may still be possible. d. Vasectomy may affect potency. e. Secondary sex characteristics are unaffected.

B, C, E Vasectomy is the most commonly used procedure for male sterilization and is performed on an outpatient basis under local anesthesia. Pain, bleeding, swelling, and infection are considered complications. Reversal is generally unsuccessful; however, it may take several weeks to months for all sperm to be cleared from the sperm ducts. Another form of contraception is necessary until the sperm counts are zero. Vasectomy has no effect on potency, and secondary sex characteristics are not affected.

Hypothyroidism occurs in 2 to 3 pregnancies per 1000. Because severe hypothyroidism is associated with infertility and miscarriage, it is not often seen in pregnancy. Regardless of this fact, the nurse should be aware of the characteristic symptoms of hypothyroidism. Which do they include? (Select all that apply.) a. Hot flashes b. Weight loss c. Lethargy d. Decrease in exercise capacity e. Cold intolerance

C, D, E Symptoms include weight gain, lethargy, decrease in exercise capacity, and intolerance to cold. Other presentations might include constipation, hoarseness, hair loss, and dry skin. Thyroid supplements are used to treat hyperthyroidism in pregnancy.

Which symptom described by a client is characteristic of premenstrual syndrome (PMS)? a. "I feel irritable and moody a week before my period is supposed to start." b. "I have lower abdominal pain beginning on the third day of my menstrual period." c. "I have nausea and headaches after my period starts, and they last 2 to 3 days." d. "I have abdominal bloating and breast pain after a couple days of my period."

a. "I feel irritable and moody a week before my period is supposed to start." PMS is a cluster of physical, psychologic, and behavioral symptoms that begin in the luteal phase of the menstrual cycle and resolve within a couple of days of the onset of menses. Complaints of lower abdominal pain, nausea and headaches, and abdominal bloating all are associated with PMS; however, the timing reflected is inaccurate.

Which statement most accurately describes the HELLP syndrome? a. Mild form of preeclampsia b. Diagnosed by a nurse alert to its symptoms c. Characterized by hemolysis, elevated liver enzymes, and low platelets d. Associated with preterm labor but not perinatal mortality

c. Characterized by hemolysis, elevated liver enzymes, and low platelets The acronym HELLP stands for hemolysis (H), elevated liver (EL) enzymes, and low platelets (LP). The HELLP syndrome is a variant of severe preeclampsia and is difficult to identify because the symptoms are not often obvious. The HELLP syndrome must be diagnosed in the laboratory. Preterm labor is greatly increased; therefore, so is perinatal mortality.

Which condition is the least likely cause of amenorrhea in a 17-year-old client? a. Anatomic abnormalities b. Type 1 diabetes mellitus c. Obesity d. Pregnancy

c. Obesity A moderately obese adolescent (20% to 30% above ideal weight) may have early onset menstruation. Girls who regularly exercise before menarche can have delayed onset of menstruation to age 18 years. Anatomic abnormalities are a possible cause of amenorrhea. Type 1 diabetes mellitus is a possible cause of amenorrhea. Pregnancy is the most common cause of amenorrhea.

Which physiologic alteration of pregnancy most significantly affects glucose metabolism? a. Pancreatic function in the islets of Langerhans is affected by pregnancy. b. Pregnant women use glucose at a more rapid rate than nonpregnant women. c. Pregnant women significantly increase their dietary intake. d. Placental hormones are antagonistic to insulin, thus resulting in insulin resistance.

d. Placental hormones are antagonistic to insulin, thus resulting in insulin resistance. Placental hormones, estrogen, progesterone, and human placental lactogen (HPL) create insulin resistance. Insulin is also broken down more quickly by the enzyme placental insulinase. Pancreatic functioning is not affected by pregnancy. The glucose requirements differ because of the growing fetus. The pregnant woman should increase her intake by 200 calories a day.

COMPLETION The practice of the calendar rhythm method is based on the number of days in each menstrual cycle. The fertile period is determined after monitoring each cycle for 6 months. The beginning of the fertile period is estimated by subtracting 18 days from the longest cycle and 11 days from the shortest. If the woman's cycles vary in length from 24 to 30 days, then her fertile period would be day _____ through day ______.

6, 19 To avoid pregnancy, the couple must abstain from intercourse on days 6 through 19. Ovulation occurs on day 12 (plus or minus 2 days either way).

The client being cared for has severe preeclampsia and is receiving a magnesium sulfate infusion. Which new finding would give the nurse cause for concern? a. Sleepy, sedated affect b. Respiratory rate of 10 breaths per minute c. Deep tendon reflexes (DTRs) of 2+ d. Absent ankle clonus

b. Respiratory rate of 10 breaths per minute A respiratory rate of 10 breaths per minute indicates the client is experiencing respiratory depression from magnesium toxicity. Because magnesium sulfate is a CNS depressant, the client will most likely become sedated when the infusion is initiated. DTRs of 2+ and absent ankle clonus are normal findings.

During an inpatient psychiatric hospitalization, what is the most important nursing intervention? a. Contacting the client's significant other b. Supervising and guiding visits with her infant c. Allowing no contact with anyone who annoys her d. Having the infant with the mother the majority of the time

b. Supervising and guiding visits with her infant In the hospital setting, the reintroduction of the infant to the mother can and should occur at the mother's own pace. A schedule is set that increases the number of hours the mother cares for her infant over several days, culminating in the infant staying overnight in the mother's room. These supervised and guided visits allow the mother to experience meeting the infant's needs and giving up sleep for the infant. Reintroducing the mother to her infant while in a supervised setting is essential. Another important task for a mother under psychiatric care is to reestablish positive interactions with others.

A client with maternal phenylketonuria (PKU) has come to the obstetrical clinic to begin prenatal care. Why would this preexisting condition result in the need for closer monitoring during pregnancy? a. PKU is a recognized cause of preterm labor. b. The fetus may develop cognitive problems. c. A pregnant woman is more likely to die without strict dietary control. d. Women with PKU are usually mentally handicapped and should not reproduce.

b. The fetus may develop cognitive problems. Children born to women with untreated PKU are more likely to be born with mental retardation, microcephaly, congenital heart disease, and low birth weight. Maternal PKU has no effect on labor. Women without dietary control of PKU are more likely to miscarry or bear a child with congenital anomalies. Screening for undiagnosed maternal PKU at the first prenatal visit may be warranted, especially in individuals with a family history of the disorder, with low intelligence of an uncertain cause, or who have given birth to microcephalic infants.

Which information should the nurse take into consideration when planning care for a postpartum client with cardiac disease? a. The plan of care for a postpartum client is the same as the plan for any pregnant woman. b. The plan of care includes rest, stool softeners, and monitoring of the effect of activity. c. The plan of care includes frequent ambulating, alternating with active range-of-motion exercises. d. The plan of care includes limiting visits with the infant to once per day.

b. The plan of care includes rest, stool softeners, and monitoring of the effect of activity. Bed rest may be ordered, with or without bathroom privileges. Bowel movements without stress or strain for the woman are promoted with stool softeners, diet, and fluids. Care of the woman with cardiac disease in the postpartum period is tailored to the woman's functional capacity. The woman will be on bed rest to conserve energy and to reduce the strain on the heart. Although the woman may need help caring for the infant, breastfeeding and infant visits are not contraindicated.

A pregnant woman is being discharged from the hospital after the placement of a cervical cerclage because of a history of recurrent pregnancy loss, secondary to an incompetent cervix. Which information regarding postprocedural care should the nurse emphasize in the discharge teaching? a. Any vaginal discharge should be immediately reported to her health care provider. b. The presence of any contractions, rupture of membranes (ROM), or severe perineal pressure should be reported. c. The client will need to arrange for care at home, because her activity level will be restricted. d. The client will be scheduled for a cesarean birth.

b. The presence of any contractions, rupture of membranes (ROM), or severe perineal pressure should be reported. Nursing care should stress the importance of monitoring for the signs and symptoms of preterm labor. Vaginal bleeding needs to be reported to her primary health care provider. Bed rest is an element of care. However, the woman may stand for periods of up to 90 minutes, which allows her the freedom to see her physician. Home uterine activity monitoring may be used to limit the woman's need for visits and to monitor her status safely at home. The cerclage can be removed at 37 weeks of gestation (to prepare for a vaginal birth), or a cesarean birth can be planned.

Which information is most important to provide to the client interested in using the lactational amenorrhea method for contraception? a. LAM is effective until the infant is 9 months of age. b. This popular method of birth control works best if the mother is exclusively breastfeeding. c. Its typical failure rate is 5%. d. Feeding intervals should be 6 hours during the day.

b. This popular method of birth control works best if the mother is exclusively breastfeeding. The LAM works best if the mother is exclusively or almost exclusively breastfeeding. Disruption of the breastfeeding pattern increases the risk of pregnancy. After the infant is 6 months of age or menstrual flow has resumed, effectiveness decreases. The typical failure rate is 1% to 2%. Feeding intervals should be no greater than 4 hours during the day and 6 hours at night.

A 26-year-old pregnant woman, gravida 2, para 1-0-0-1, is 28 weeks pregnant when she experiences bright red, painless vaginal bleeding. On her arrival at the hospital, which diagnostic procedure will the client most likely have performed? a. Amniocentesis for fetal lung maturity b. Transvaginal ultrasound for placental location c. Contraction stress test (CST) d. Internal fetal monitoring

b. Transvaginal ultrasound for placental location The presence of painless bleeding should always alert the health care team to the possibility of placenta previa, which can be confirmed through ultrasonography. Amniocentesis is not performed on a woman who is experiencing bleeding. In the event of an imminent delivery, the fetus is presumed to have immature lungs at this gestational age, and the mother is given corticosteroids to aid in fetal lung maturity. A CST is not performed at a preterm gestational age. Furthermore, bleeding is a contraindication to a CST. Internal fetal monitoring is also contraindicated in the presence of bleeding.

In terms of the incidence and classification of diabetes, which information should the nurse keep in mind when evaluating clients during their ongoing prenatal appointments? a. Type 1 diabetes is most common. b. Type 2 diabetes often goes undiagnosed. c. Gestational diabetes mellitus (GDM) means that the woman will receive insulin treatment until 6 weeks after birth. d. Type 1 diabetes may become type 2 during pregnancy.

b. Type 2 diabetes often goes undiagnosed. Type 2 diabetes often goes undiagnosed because hyperglycemia gradually develops and is often not severe. Type 2, sometimes called adult-onset diabetes, is the most common type of diabetes. GDM refers to any degree of glucose intolerance first recognized during pregnancy; insulin may or may not be needed. People do not go back and forth between type 1 and type 2 diabetes.

Bell palsy is an acute idiopathic facial paralysis, the cause for which remains unknown. Which statement regarding this condition is correct? a. Bell palsy is the sudden development of bilateral facial weakness. b. Women with Bell palsy have an increased risk for hypertension. c. Pregnant women are affected twice as often as nonpregnant women. d. Bell palsy occurs most frequently in the first trimester.

b. Women with Bell palsy have an increased risk for hypertension. The clinical manifestations of Bell palsy include the development of unilateral facial weakness, pain surrounding the ears, difficulty closing the eye, and hyperacusis. The cause is unknown; however, Bell palsy may be related to a viral infection. Pregnant women are affected at a rate of three to five times that of nonpregnant women. The incidence rate peaks during the third trimester and puerperium. Women who develop Bell palsy in pregnancy have an increased risk for hypertension.

Nafarelin is used to treat mild-to-severe endometriosis. What instruction or information should the nurse provide to a client regarding nafarelin administration? a. Nafarelin stimulates the secretion of gonadotropin-releasing hormone (GnRH), thereby stimulating ovarian activity. b. It should be administered by intramuscular (IM) injection. c. Nafarelin should be administered by a subcutaneous implant. d. It can cause the client to experience some hot flashes and vaginal dryness.

d. It can cause the client to experience some hot flashes and vaginal dryness. Nafarelin is a GnRH agonist, and its side effects are similar to those of menopause. The hypoestrogenism effect results in hot flashes and vaginal dryness. Nafarelin is a GnRH agonist that suppresses the secretion of GnRH. Nafarelin is administered twice daily by nasal spray and can be intranasally administered. Leuprolide is given once per month by IM injection. Goserelin is administered by subcutaneous implant.

The nurse has evaluated a client with preeclampsia by assessing deep tendon reflexes (DTRs). The result is a grade of 3+. Which DTR response most accurately describes this score? a. Sluggish or diminished b. Brisk, hyperactive, with intermittent or transient clonus c. Active or expected response d. More brisk than expected, slightly hyperactive

d. More brisk than expected, slightly hyperactive DTRs reflect the balance between the cerebral cortex and the spinal cord. They are evaluated at baseline and to detect changes. A slightly hyperactive and brisk response indicates a grade 3+ response.

A woman at 39 weeks of gestation with a history of preeclampsia is admitted to the labor and birth unit. She suddenly experiences increased contraction frequency of every 1 to 2 minutes, dark red vaginal bleeding, and a tense, painful abdomen. Which clinical change does the nurse anticipate? a. Eclamptic seizure b. Rupture of the uterus c. Placenta previa d. Placental abruption

d. Placental abruption Uterine tenderness in the presence of increasing tone may be the earliest sign of placental abruption. Women with preeclampsia are at increased risk for an abruption attributable to decreased placental perfusion. Eclamptic seizures are evidenced by the presence of generalized tonic-clonic convulsions. Uterine rupture exhibits hypotonic uterine activity, signs of hypovolemia, and, in many cases, the absence of pain. Placenta previa exhibits bright red, painless vaginal bleeding.

During a prenatal visit, the nurse is explaining dietary management to a woman diagnosed with pre-gestational diabetes. Which statement by the client reassures the nurse that teaching has been effective? a. "I will need to eat 600 more calories per day because I am pregnant." b. "I can continue with the same diet as before pregnancy as long as it is well balanced." c. "Diet and insulin needs change during pregnancy." d. "I will plan my diet based on the results of urine glucose testing."

c. "Diet and insulin needs change during pregnancy." Diet and insulin needs change during the pregnancy in direct correlation to hormonal changes and energy needs. In the third trimester, insulin needs may double or even quadruple. The diet is individualized to allow for increased fetal and metabolic requirements, with consideration of such factors as prepregnancy weight and dietary habits, overall health, ethnic background, lifestyle, stage of pregnancy, knowledge of nutrition, and insulin therapy. Energy needs are usually calculated on the basis of 30 to 35 calories per kilogram of ideal body weight. Dietary management during a diabetic pregnancy must be based on blood, not urine, glucose changes.

A woman with worsening preeclampsia is admitted to the hospital's labor and birth unit. The physician explains the plan of care for severe preeclampsia, including the induction of labor, to the woman and her partner. Which statement by the partner leads the nurse to believe that the couple needs further information? a. "I will help her use the breathing techniques that we learned in our childbirth classes." b. "I will give her ice chips to eat during labor." c. "Since we will be here for a while, I'll ask my mother, to bring our toddler to visit." d. "I will stay with her during her labor, just as we planned."

c. "Since we will be here for a while, I'll ask my mother, to bring our toddler to visit." Arranging a visit with their toddler indicates that the partner does not understand the importance of the quiet, subdued environment that is needed to prevent this condition from worsening. Implementing breathing techniques is indicative of adequate knowledge related to pain management during labor. Administering ice chips indicates an understanding of nutritional needs during labor. Staying with his partner during labor demonstrates the husband's support and is appropriate.

A woman with asthma is experiencing a postpartum hemorrhage. Which drug should be avoided when treating postpartum bleeding to avoid exacerbating asthma? a. Oxytocin (Pitocin) b. Nonsteroidal antiinflammatory drugs (NSAIDs) c. 15-methyl PGF2(alpha symbol) d. Fentanyl

c. 15-methyl PGF2(alpha symbol) Prostaglandin derivatives like 15-methyl PGF2 should not be used to treat women with asthma, because they may exacerbate symptoms. Oxytocin is the drug of choice to treat this woman's bleeding; it will not exacerbate her asthma. NSAIDs are not used to treat bleeding. Fentanyl is used to treat pain, not bleeding.

Which client exhibits the greatest number of risk factors associated with the development of preeclampsia? a. 30-year-old obese Caucasian with her third pregnancy b. 41-year-old Caucasian primigravida c. 19-year-old African American who is pregnant with twins d. 25-year-old Asian American whose pregnancy is the result of donor insemination

c. 19-year-old African American who is pregnant with twins Three risk factors are present in the 19-year-old African-American client. She has African-American ethnicity, is at the young end of the age distribution, and has a multiple pregnancy. In planning care for this client, the nurse must frequently monitor her BP and teach her to recognize the early warning signs of preeclampsia. The 30-year-old obese Caucasian client has only has one known risk factor: obesity. Age distribution appears to be U-shaped, with women younger than 20 years of age and women older than 40 years of age being at greatest risk. Preeclampsia continues to be more frequently observed in primigravidas; this client is a multigravida woman. Two risk factors are present for the 41-year-old Caucasian primigravida client. Her age and status as a primigravida place her at increased risk for preeclampsia. Caucasian women are at a lower risk than are African-American women. The 25-year-old Asian-American client exhibits only one risk factor. Pregnancies that result from donor insemination, oocyte donation, and embryo donation are at an increased risk of developing preeclampsia.

In the acronym BRAIDED, which letter is used to identify the key components of informed consent that the nurse must document? a. B stands for birth control. b. R stands for reproduction. c. A stands for alternatives. d. I stands for ineffective.

c. A stands for alternatives. In the acronym BRAIDED, A stands for alternatives and information about other viable methods. B stands for benefits and information about the advantages of a particular birth control method and its success rates. R stands for risks and information about the disadvantages of a particular method and its failure rates. I stands for inquiries and the opportunity to ask questions.

A woman arrives for evaluation of signs and symptoms that include a missed period, adnexal fullness, tenderness, and dark red vaginal bleeding. On examination, the nurse notices an ecchymotic blueness around the woman's umbilicus. What does this finding indicate? a. Normal integumentary changes associated with pregnancy b. Turner sign associated with appendicitis c. Cullen sign associated with a ruptured ectopic pregnancy d. Chadwick sign associated with early pregnancy

c. Cullen sign associated with a ruptured ectopic pregnancy Cullen sign, the blue ecchymosis observed in the umbilical area, indicates hematoperitoneum associated with an undiagnosed ruptured intraabdominal ectopic pregnancy. Linea nigra on the abdomen is the normal integumentary change associated with pregnancy and exhibits a brown pigmented, vertical line on the lower abdomen. Turner sign is ecchymosis in the flank area, often associated with pancreatitis. A Chadwick sign is a blue-purple cervix that may be seen during or around the eighth week of pregnancy.

A primigravida is being monitored at the prenatal clinic for preeclampsia. Which finding is of greatest concern to the nurse? a. Blood pressure (BP) increase to 138/86 mm Hg b. Weight gain of 0.5 kg during the past 2 weeks c. Dipstick value of 3+ for protein in her urine d. Pitting pedal edema at the end of the day

c. Dipstick value of 3+ for protein in her urine Proteinuria is defined as a concentration of 1+ or greater via dipstick measurement. A dipstick value of 3+ alerts the nurse that additional testing or assessment should be performed. A 24-hour urine collection is preferred over dipstick testing attributable to accuracy. Generally, hypertension is defined as a BP of 140/90 mm Hg or an increase in systolic pressure of 30 mm Hg or diastolic pressure of 15 mm Hg. Preeclampsia may be demonstrated as a rapid weight gain of more than 2 kg in 1 week. Edema occurs in many normal pregnancies, as well as in women with preeclampsia. Therefore, the presence of edema is no longer considered diagnostic of preeclampsia.

Which is the most accurate description of postpartum depression (PPD) without psychotic features? a. Postpartum baby blues requiring the woman to visit with a counselor or psychologist b. Condition that is more common among older Caucasian women because they have higher expectations c. Distinguishable by pervasive sadness along with mood swings d. Condition that disappears without outside help

c. Distinguishable by pervasive sadness along with mood swings PPD is characterized by an intense pervasive sadness along with labile mood swings and is more persistent than postpartum baby blues. PPD, even without psychotic features, is more serious and persistent than postpartum baby blues. PPD is more common among younger mothers and African-American mothers. Most women need professional help to get through PPD, including pharmacologic intervention.

Several metabolic changes occur throughout pregnancy. Which physiologic adaptation of pregnancy will influence the nurse's plan of care? a. Insulin crosses the placenta to the fetus only in the first trimester, after which the fetus secretes its own. b. Women with insulin-dependent diabetes are prone to hyperglycemia during the first trimester because they are consuming more sugar. c. During the second and third trimesters, pregnancy exerts a diabetogenic effect that ensures an abundant supply of glucose for the fetus. d. Maternal insulin requirements steadily decline during pregnancy.

c. During the second and third trimesters, pregnancy exerts a diabetogenic effect that ensures an abundant supply of glucose for the fetus. Pregnant women develop increased insulin resistance during the second and third trimesters. Insulin never crosses the placenta; the fetus starts making its own around the 10th week. As a result of normal metabolic changes during pregnancy, insulin-dependent women are prone to hypoglycemia (low levels). Maternal insulin requirements may double or quadruple by the end of pregnancy.

To manage her diabetes appropriately and to ensure a good fetal outcome, how would the pregnant woman with diabetes alter her diet? a. Eat six small equal meals per day. b. Reduce the carbohydrates in her diet. c. Eat her meals and snacks on a fixed schedule. d. Increase her consumption of protein.

c. Eat her meals and snacks on a fixed schedule. Having a fixed meal schedule will provide the woman and the fetus with a steady blood sugar level, provide a good balance with insulin administration, and help prevent complications. Having a fixed meal schedule is more important than the equal division of food intake. Approximately 45% of the food eaten should be in the form of carbohydrates.

Which statement related to the condition of endometriosis accurately describes it's pathophysiology? a. Endometriosis is characterized by the presence and growth of endometrial tissue inside the uterus. b. It is found more often in African-American women than in Caucasian or Asian women. c. Endometriosis may worsen with repeated cycles or remain asymptomatic and disappear after menopause. d. It is unlikely to affect sexual intercourse or fertility.

c. Endometriosis may worsen with repeated cycles or remain asymptomatic and disappear after menopause. With endometriosis, the endometrial tissue is outside the uterus. Endometriosis is found equally in Caucasian and African-American women and is slightly more prevalent in Asian women. Symptoms vary among women, ranging from nonexistent to incapacitating. The condition is seven times more prevalent in women who have a first-degree relative with endometriosis. Women can experience painful intercourse and impaired fertility with endometriosis.

Which neurologic condition would require preconception counseling, if possible? a. Eclampsia b. Bell palsy c. Epilepsy d. Multiple sclerosis

c. Epilepsy Women with epilepsy should receive preconception counseling, if at all possible. Achieving seizure control before becoming pregnant is a desirable state. Medication should also be carefully reviewed. Eclampsia may sometimes be confused with epilepsy, and Bell palsy is a form of facial paralysis; preconception counseling for either condition is not essential to care. Multiple sclerosis is a patchy demyelination of the spinal cord that does not affect the normal course of pregnancy or birth.

Which finding on a prenatal visit at 10 weeks of gestation might suggest a hydatidiform mole? a. Complaint of frequent mild nausea b. Blood pressure of 120/80 mm Hg c. Fundal height measurement of 18 cm d. History of bright red spotting for 1 day, weeks ago

c. Fundal height measurement of 18 cm The uterus in a hydatidiform molar pregnancy is often larger than would be expected on the basis of the duration of the pregnancy. Nausea increases in a molar pregnancy because of the increased production of hCG. A woman with a molar pregnancy may have early-onset pregnancy-induced hypertension. In the client's history, bleeding is normally described as brownish.

What condition indicates concealed hemorrhage when the client experiences abruptio placentae? a. Decrease in abdominal pain b. Bradycardia c. Hard, boardlike abdomen d. Decrease in fundal height

c. Hard, boardlike abdomen Concealed hemorrhage occurs when the edges of the placenta do not separate. The formation of a hematoma behind the placenta and subsequent infiltration of the blood into the uterine muscle results in a very firm, boardlike abdomen. Abdominal pain may increase. The client will have shock symptoms that include tachycardia. As bleeding occurs, the fundal height increases.

When a woman is diagnosed with postpartum depression (PPD) with psychotic features, what is the nurse's primary concern in planning the client's care? a. Displaying outbursts of anger b. Neglecting her hygiene c. Harming her infant d. Losing interest in her husband

c. Harming her infant Thoughts of harm to herself or to the infant are among the most serious symptoms of PPD and require immediate assessment and intervention. Although outbursts of anger and neglecting personal hygiene are symptoms attributable to PPD, the major concern remains the potential of harm to herself or her infant. Although this client is likely to lose interest in her spouse, it is not the nurse's primary concern.

The labor of a pregnant woman with preeclampsia is going to be induced. Before initiating the oxytocin infusion, the nurse reviews the woman's latest laboratory test findings, which reveal a platelet count of 90,000 mm3, an elevated aspartate aminotransaminase (AST) level, and a falling hematocrit. The laboratory results are indicative of which condition? a. Eclampsia b. Disseminated intravascular coagulation (DIC) syndrome c. Hemolysis, elevated liver enzyme levels, and low platelet levels (HELLP) syndrome d. Idiopathic thrombocytopenia

c. Hemolysis, elevated liver enzyme levels, and low platelet levels (HELLP) syndrome HELLP syndrome is a laboratory diagnosis for a variant of severe preeclampsia that involves hepatic dysfunction characterized by hemolysis (H), elevated liver (EL) enzymes, and low platelets (LP). Eclampsia is determined by the presence of seizures. DIC is a potential complication associated with HELLP syndrome. Idiopathic thrombocytopenia is the presence of low platelets of unknown cause and is not associated with preeclampsia.

Which important component of nutritional counseling should the nurse include in health teaching for a pregnant woman who is experiencing cholecystitis? a. Assess the woman's dietary history for adequate calories and proteins. b. Teach the woman that the bulk of calories should come from proteins. c. Instruct the woman to eat a low-fat diet and to avoid fried foods. d. Instruct the woman to eat a low-cholesterol, low-salt diet.

c. Instruct the woman to eat a low-fat diet and to avoid fried foods. Eating a low-fat diet and avoiding fried foods is appropriate nutritional counseling for this client. Caloric and protein intake do not predispose a woman to the development of cholecystitis. The woman should be instructed to limit protein intake and choose foods that are high in carbohydrates. A low-cholesterol diet may be the result of limiting fats. However, a low-salt diet is not indicated.

The management of the pregnant client who has experienced a pregnancy loss depends on the type of miscarriage and the signs and symptoms. While planning care for a client who desires outpatient management after a first-trimester loss, what would the nurse expect the plan to include? a. Dilation and curettage (D&C) b. Dilation and evacuation (D&E) c. Misoprostol d. Ergot products

c. Misoprostol Outpatient management of a first-trimester loss is safely accomplished by the intravaginal use of misoprostol for up to 2 days. If the bleeding is uncontrollable, vital signs are unstable, or signs of infection are present, then a surgical evacuation should be performed. D&C is a surgical procedure that requires dilation of the cervix and scraping of the uterine walls to remove the contents of pregnancy. This procedure is commonly performed to treat inevitable or incomplete abortion and should be performed in a hospital. D&E is usually performed after 16 weeks of pregnancy. The cervix is widely dilated, followed by removal of the contents of the uterus. Ergot products such as Methergine or Hemabate may be administered for excessive bleeding after miscarriage.

Which pharmacologic therapy provides optimal pain relief for primary dysmenorrhea? a. Acetaminophen b. Oral contraceptive pills (OCPs) c. Nonsteroidal antiinflammatory drugs (NSAIDs) d. Aspirin

c. Nonsteroidal antiinflammatory drugs (NSAIDs) NSAIDs have the strongest research results for pain relief. If one NSAID is not effective, then another one may provide relief. Approximately 80% of women find relief from these prostaglandin inhibitors. Preparations containing acetaminophen are less effective for dysmenorrhea because they lack the antiprostaglandin properties of NSAIDs. OCPs are a reasonable choice for women who also want birth control. The benefit of OCPs is the reduction of menstrual flow and irregularities. OCPs may be contraindicated for some women and have a number of potential side effects. NSAIDs are the drug of choice. However, if a woman is taking an NSAID, she should avoid taking aspirin as well.

While providing care to the maternity client, the nurse should be aware that one of these anxiety disorders is likely to be triggered by the process of labor and birth. Which disorder fits this criterion? a. Phobias b. Panic disorder c. Posttraumatic stress disorder (PTSD) d. Obsessive-compulsive disorder (OCD)

c. Posttraumatic stress disorder (PTSD) PTSD can occur as the result of a past trauma such as rape. Symptoms of PTSD include re-experiencing the event, numbing, irritability, angry outbursts, and exaggerated startle reflex. With the increased bodily touch and vaginal examinations that occur during labor, the client may have memories of the original trauma. The process of giving birth may result in her feeling out of control. The nurse should verbalize an understanding and reassure the client as necessary. Phobias are irrational fears that may lead a person to avoid certain events or situations. Panic disorders may occur in as many as 3% to 5% of women in the postpartum period and are described as episodes of intense apprehension, fear, and terror. Symptoms of a panic disorder may include palpitations, chest pain, choking, or smothering. OCD symptoms include recurrent, persistent, and intrusive thoughts. The mother may repeatedly check and recheck her infant once he or she is born, although she realizes that this behavior is irrational. OCD is optimally treated with medications.

Which preexisting factor is known to increase the risk of gestational diabetes mellitus (GDM)? a. Underweight before pregnancy b. Maternal age younger than 25 years c. Previous birth of large infant d. Previous diagnosis of type 2 diabetes mellitus

c. Previous birth of large infant A previous birth of a large infant suggests GDM. Obesity (body mass index [BMI] of 30 or greater) creates a higher risk for gestational diabetes. A woman younger than 25 years is not generally at risk for GDM. The person with type 2 diabetes mellitus already has diabetes and thus will continue to have it after pregnancy. Insulin may be required during pregnancy because oral hypoglycemia drugs are contraindicated during pregnancy.

A pregnant woman who abuses cocaine admits to exchanging sex to finance her drug habit. This behavior places the client at the greatest risk for what? a. Depression of the CNS b. Hypotension and vasodilation c. Sexually transmitted infections (STIs) d. Postmature birth

c. Sexually transmitted infections (STIs) Exchanging sex acts for drugs places the woman at increased risk for STIs because of multiple partners and the lack of protection. Cocaine is a CNS stimulant that causes HTN and vasoconstriction. Premature delivery of the infant is one of the more common problems associated with cocaine use during pregnancy.

A woman who is 30 weeks of gestation arrives at the hospital with bleeding. Which differential diagnosis would not be applicable for this client? a. Placenta previa b. Abruptio placentae c. Spontaneous abortion d. Cord insertion

c. Spontaneous abortion Spontaneous abortion is another name for miscarriage; it occurs, by definition, early in pregnancy. Placenta previa is a well-known reason for bleeding late in pregnancy. The premature separation of the placenta (abruptio placentae) is a bleeding disorder that can occur late in pregnancy. Cord insertion may cause a bleeding disorder that can also occur late in pregnancy.

A pregnant woman at term is transported to the emergency department (ED) after a severe vehicular accident. The obstetric nurse responds and rushes to the ED with a fetal monitor. Cardiopulmonary arrest occurs as the obstetric nurse arrives. What is the highest priority for the trauma team? a. Obtaining IV access, and starting aggressive fluid resuscitation b. Quickly applying the fetal monitor to determine whether the fetus viability c. Starting cardiopulmonary resuscitation (CPR) d. Transferring the woman to the surgical unit for an emergency cesarean delivery in case the fetus is still alive

c. Starting cardiopulmonary resuscitation (CPR) In a situation of severe maternal trauma, the systematic evaluation begins with a primary survey and the initial ABCs (airway, breathing, and circulation) of resuscitation. CPR is initiated first, followed by intravenous (IV) replacement fluid. After immediate resuscitation and successful stabilization measures, a more detailed secondary survey of the mother and fetus should be accomplished. Attempts at maternal resuscitation are made, followed by a secondary survey of the fetus. In the presence of multisystem trauma, a cesarean delivery may be indicated to increase the chance for maternal survival.

A woman will be taking oral contraceptives using a 28-day pack. What advice should the nurse provide to protect this client from an unintended pregnancy? a. Limit sexual contact for one cycle after starting the pill. b. Use condoms and foam instead of the pill for as long as the client takes an antibiotic. c. Take one pill at the same time every day. d. Throw away the pack and use a backup method if two pills are missed during week 1 of her cycle.

c. Take one pill at the same time every day. To maintain adequate hormone levels for contraception and to enhance compliance, clients should take oral contraceptives at the same time each day. If contraceptives are to be started at any time other than during normal menses or within 3 weeks after birth or an abortion, then another method of contraception should be used through the first week to prevent the risk of pregnancy. Taken exactly as directed, oral contraceptives prevent ovulation, and pregnancy cannot occur. No strong pharmacokinetic evidence indicates a link between the use of broad-spectrum antibiotics and altered hormonal levels in oral contraceptive users. If the client misses two pills during week 1, then she should take two pills a day for 2 days and finish the package and use a backup contraceptive method for the next 7 consecutive days.

Which client would be an ideal candidate for injectable progestins such as medroxyprogesterone acetate as a contraceptive choice? a. The ideal candidate wants menstrual regularity and predictability. b. The client has a history of thrombotic problems or breast cancer. c. The ideal candidate has difficulty remembering to take oral contraceptives daily. d. The client is homeless or mobile and rarely receives health care.

c. The ideal candidate has difficulty remembering to take oral contraceptives daily. Advantages of medroxyprogesterone acetate includes its contraceptive effectiveness, compared with the effectiveness of combined oral contraceptives, and the requirement of only four injections a year. The disadvantages of injectable progestins are prolonged amenorrhea and uterine bleeding. The use of injectable progestin carries an increased risk of venous thrombosis and thromboembolism. To be effective, injections must be administered every 11 to 13 weeks. Access to health care is necessary to prevent pregnancy or potential complications.

Which statement regarding the laboratory test for glycosylated hemoglobin Alc is correct? a. The laboratory test for glycosylated hemoglobin Alc is performed for all pregnant women, not only those with or likely to have diabetes. b. This laboratory test is a snapshot of glucose control at the moment. c. This laboratory test measures the levels of hemoglobin Alc, which should remain at less than 7%. d. This laboratory test is performed on the woman's urine, not her blood.

c. This laboratory test measures the levels of hemoglobin Alc, which should remain at less than 7%. Hemoglobin Alc levels greater than 7% indicate an elevated glucose level during the previous 4 to 6 weeks. This extra laboratory test is for diabetic women and defines glycemic control over the previous 4 to 6 weeks. Glycosylated hemoglobin level tests are performed on the blood.

The nurse is providing contraceptive instruction to a young couple who are eager to learn. The nurse should be cognizant of which information regarding the natural family planning method? a. The natural family planning method is the same as coitus interruptus or "pulling out." b. This contraception method uses the calendar method to align the woman's cycle with the natural phases of the moon. c. This practice is the only contraceptive method acceptable to the Roman Catholic Church. d. The natural family planning method relies on barrier methods during the fertility phases.

c. This practice is the only contraceptive method acceptable to the Roman Catholic Church. Natural family planning is the only contraceptive practice acceptable to the Roman Catholic Church. "Pulling out" is not the same as periodic abstinence, another name for natural family planning. The phases of the moon are not part of the calendar method or any method. Natural family planning is another name for periodic abstinence, which is the accepted way to pass safely through the fertility phases without relying on chemical or physical barriers.

A woman arrives at the emergency department with reports of bleeding and cramping. The initial nursing history is significant for a last menstrual period 6 weeks ago. On sterile speculum examination, the primary care provider finds that the cervix is closed. The anticipated plan of care for this woman would be based on a probable diagnosis of which type of spontaneous abortion? a. Incomplete b. Inevitable c. Threatened d. Septic

c. Threatened A woman with a threatened abortion has spotting, mild cramps, and no cervical dilation. A woman with an incomplete abortion would have heavy bleeding, mild-to-severe cramping, and cervical dilation. An inevitable abortion demonstrates the same symptoms as an incomplete abortion: heavy bleeding, mild-to-severe cramping, and cervical dilation. A woman with a septic abortion has malodorous bleeding and typically a dilated cervix.

A pregnant woman has been receiving a magnesium sulfate infusion for treatment of severe preeclampsia for 24 hours. On assessment, the nurse finds the following vital signs: temperature 37.3° C, pulse rate 88 beats per minute, respiratory rate 10 breaths per minute, BP 148/90 mm Hg, absent deep tendon reflexes (DTRs), and no ankle clonus. The client complains, "I'm so thirsty and warm." What is the nurse's immediate action? a. To call for an immediate magnesium sulfate level b. To administer oxygen c. To discontinue the magnesium sulfate infusion d. To prepare to administer hydralazine

c. To discontinue the magnesium sulfate infusion Regardless of the magnesium level, the client is displaying the clinical signs and symptoms of magnesium toxicity. The first action by the nurse should be to discontinue the infusion of magnesium sulfate. In addition, calcium gluconate, the antidote for magnesium, may be administered. Hydralazine is an antihypertensive drug commonly used to treat hypertension in severe preeclampsia. Typically, hydralazine is administered for a systolic BP higher than 160 mm Hg or a diastolic BP higher than 110 mm Hg.

The nurse is preparing to administer methotrexate to the client. This hazardous drug is most often used for which obstetric complication? a. Complete hydatidiform mole b. Missed abortion c. Unruptured ectopic pregnancy d. Abruptio placentae

c. Unruptured ectopic pregnancy Methotrexate is an effective nonsurgical treatment option for a hemodynamically stable woman whose ectopic pregnancy is unruptured and measures less than 4 cm in diameter. Methotrexate is not indicated or recommended as a treatment option for a complete hydatidiform mole, for a missed abortion, or for abruptio placentae.

A client currently uses a diaphragm and spermicide for contraception. She asks the nurse to explain the major differences between the cervical cap and the diaphragm. What is the most appropriate response by the nurse? a. "No spermicide is used with the cervical cap, so it's less messy." b. "The diaphragm can be left in place longer after intercourse." c. "Repeated intercourse with the diaphragm is more convenient." d. "The cervical cap can be safely used for repeated acts of intercourse without adding more spermicide later."

d. "The cervical cap can be safely used for repeated acts of intercourse without adding more spermicide later." The cervical cap can be inserted hours before sexual intercourse without the need for additional spermicide later. Spermicide should be used inside the cap as an additional chemical barrier. The cervical cap should remain in place for 6 hours after the last act of intercourse. Repeated intercourse with the cervical cap is more convenient because no additional spermicide is needed.

A woman is using the basal body temperature (BBT) method of contraception. She calls the clinic and tells the nurse, "My period is due in a few days, and my temperature has not gone up." What is the nurse's most appropriate response? a. "This probably means that you're pregnant." b. "Don't worry; it's probably nothing." c. "Have you been sick this month?" d. "You probably didn't ovulate during this cycle."

d. "You probably didn't ovulate during this cycle." The absence of a temperature decrease most likely is the result of a lack of ovulation. Pregnancy cannot occur without ovulation, which is being measured using the BBT method. A comment such as, "Don't worry; it's probably nothing," discredits the client's concerns. Illness is most likely the cause of an increase in BBT.

A woman diagnosed with gestational diabetes has had little or no experience reading and interpreting glucose levels. The client shows the nurse her readings for the past few days. Which reading signals the nurse that the client may require an adjustment of insulin or carbohydrates? a. 75 mg/dl before lunch. This is low; better eat now. b. 115 mg/dl 1 hour after lunch. This is a little high; maybe eat a little less next time. c. 115 mg/dl 2 hours after lunch. This is too high; it is time for insulin. d. 50 mg/dl just after waking up from a nap. This is too low; maybe eat a snack before going to sleep.

d. 50 mg/dl just after waking up from a nap. This is too low; maybe eat a snack before going to sleep. 50 mg/dl after waking from a nap is too low. During hours of sleep, glucose levels should not be less than 60 mg/dl. Snacks before sleeping can be helpful. The premeal acceptable range is 60 to 99 mg/dl. The readings 1 hour after a meal should be less than 129 mg/dl. Two hours after eating, the readings should be less than 120 mg/dl.

Which statement best describes chronic hypertension? a. Chronic hypertension is defined as hypertension that begins during pregnancy and lasts for the duration of the pregnancy. b. Chronic hypertension is considered severe when the systolic BP is higher than 140 mm Hg or the diastolic BP is higher than 90 mm Hg. c. Chronic hypertension is general hypertension plus proteinuria. d. Chronic hypertension can occur independently of or simultaneously with preeclampsia.

d. Chronic hypertension can occur independently of or simultaneously with preeclampsia. Women with chronic hypertension may develop superimposed preeclampsia, which increases the morbidity for both the mother and the fetus. Chronic hypertension is present before pregnancy or diagnosed before the 20 weeks of gestation and persists longer than 6 weeks postpartum. Chronic hypertension becomes severe with a diastolic BP of 110 mm Hg or higher. Proteinuria is an excessive concentration of protein in the urine and is a complication of hypertension, not a defining characteristic.

As a powerful central nervous system (CNS) stimulant, which of these substances can lead to miscarriage, preterm labor, placental separation (abruption), and stillbirth? a. Heroin b. Alcohol c. Phencyclidine (1 phenylcyclohexylpiperidine; PCP) d. Cocaine

d. Cocaine Cocaine is a powerful CNS stimulant. Effects on pregnancy associated with cocaine use include abruptio placentae, preterm labor, precipitous birth, and stillbirth. Heroin is an opiate; its use in pregnancy is associated with preeclampsia, intrauterine growth restriction, miscarriage, premature rupture of membranes, infections, breech presentation, and preterm labor. The most serious effect of alcohol use in pregnancy is FAS. The major concern regarding PCP use in pregnant women is its association with polydrug abuse and its neurobehavioral effects on the neonate.

When caring for a pregnant woman with cardiac problems, the nurse must be alert for the signs and symptoms of cardiac decompensation. Which critical findings would the nurse find on assessment of the client experiencing this condition? a. Regular heart rate and hypertension b. Increased urinary output, tachycardia, and dry cough c. Shortness of breath, bradycardia, and hypertension d. Edema, crackles, and cyanosis of nails and lips

d. Edema, crackles, and cyanosis of nails and lips Signs of cardiac decompensation include dyspnea; crackles; an irregular, weak, and rapid pulse; rapid respirations; a moist and frequent cough; generalized edema; increasing fatigue; and cyanosis of the lips and nailbeds. A regular heart rate and hypertension are not generally associated with cardiac decompensation. Of the symptoms of increased urinary output, tachycardia, and dry cough, only tachycardia is indicative of cardiac decompensation. Of the symptoms of shortness of breath, bradycardia, and hypertension, only dyspnea is indicative of cardiac decompensation.

A woman who has undergone an induced abortion be instructed to return to the emergency department when what situation exists? (Select all that apply.) a. Fever higher than 38 C b. Chills c. Foul-smelling vaginal discharge d. Bleeding greater than 2 pads in 2 hours e. Abdominal tenderness and or pain

A, B, C, D, E The client should report to a health care facility for any of the following symptoms: fever higher than 38 C, chills, bleeding more than two saturated pads in 2 hours or heavy bleeding lasting for days, foul-smelling discharge, abdominal tenderness or pain, and cramping or backache.

Screening questions for alcohol and drug abuse should be included in the overall assessment during the first prenatal visit for all women. The 4 Ps Plus is a screening tool specifically designed to identify the need for a more in-depth assessment. Which are the correct components of the 4 Ps Plus? (Select all that apply.) a. Parents b. Partner c. Present d. Past e. Pregnancy

A, B, D, E The nurse who is screening the client using the 4 Ps Plus would use the following format: Parents: "Did either of your parents have a problem with alcohol or drugs?" Partner: "Does your partner have a problem with alcohol or drugs?" Past: "Have you ever had any beer, wine, or liquor?" Pregnancy: "In the month before you knew you were pregnant, how many cigarettes did you smoke? How much beer, wine, or liquor did you drink?" Present: Is not a component of the 4 Ps Plus.

Nurses are in an ideal position to educate clients who experience Premenstrual dysphoric disorder (PMDD). What self-help activities have been documented as helpful in alleviating the symptoms of PMDD? (Select all that apply.) a. Regular exercise b. Improved nutrition c. Daily glass of wine d. Smoking cessation e. Oil of evening primrose

A, B, D, E Regular exercise, improved nutrition, smoking cessation, and oil of evening primrose are accurate modalities that may provide significant symptom relief in 1 to 2 months. If no improvement is realized after these changes have been made, then the client may need to begin pharmacologic therapy. Women should decrease their alcohol and caffeinated beverage consumption if they suffer from PMDD.

Diabetes refers to a group of metabolic diseases characterized by hyperglycemia resulting from defects in insulin action, insulin secretion, or both. Over time, diabetes causes significant changes in the microvascular and macrovascular circulations. What do these complications include? (Select all that apply.) a. Atherosclerosis b. Retinopathy c. Intrauterine fetal death (IUFD) d. Nephropathy e. Neuropathy f. Autonomic neuropathy

A, B, D, E These structural changes will most likely affect a variety of systems, including the heart, eyes, kidneys, and nerves. IUFD (stillbirth) remains a major complication of diabetes in pregnancy; however, this is a fetal complication.

One of the most important components of the physical assessment of the pregnant client is the determination of blood pressure (BP). Which techniques are important in obtaining accurate BP readings? (Select all that apply.) a. The client should be seated. b. The client's arm should be placed at the level of the heart. c. An electronic BP device should be used. d. The cuff should cover a minimum of 60% of the upper arm. e. The same arm should be used for every reading.

A, B, E BP readings are easily affected by maternal position. Ideally, the client should be seated. An alternative position is left lateral recumbent with the arm at the level of the heart. The arm should always be held in a horizontal position at approximately the level of the heart. The same arm should be used at every visit. The manual sphygmomanometer is the most accurate device. If manual and electronic devices are used in the care setting, then the nurse must use caution when interpreting the readings. A proper size cuff should cover at least 80% of the upper arm or be approximately 1.5 times the length of the upper arm

A serious but uncommon complication of undiagnosed or partially treated hyperthyroidism is a thyroid storm, which may occur in response to stress such as infection, birth, or surgery. What are the signs and symptoms of this emergency disorder? (Select all that apply.) a. Fever b. Hypothermia c. Restlessness d. Bradycardia e. Hypertension

A, C Fever, restlessness, tachycardia, vomiting, hypotension, and stupor are symptoms of a thyroid storm. Fever, not hypothermia; tachycardia, not bradycardia; and hypotension, not hypertension, are symptoms of thyroid storm.

Which suggestions are appropriate for a client who reports experiencing hot flashes? (Select all that apply.) a. Avoiding caffeine. b. Drinking a glass of wine to relax. c. Wearing layered clothing. d. Drinking ice water. e. Consuming spicy foods

A, C, D Layered clothing allows the client to remove layers if a hot flash occurs. Ice water may help alleviate the hot flashes. Slow, deep breathing is also beneficial. Avoid triggers such as exercising on hot days, spicy foods, hot beverages, and alcohol.

In caring for a pregnant woman diagnosed with sickle cell anemia, the nurse must be aware of the signs and symptoms of a sickle cell crisis. What do these include? (Select all that apply.) a. Fever b. Endometritis c. Abdominal pain d. Joint pain e. Urinary tract infection (UTI)

A, C, D Women with sickle cell anemia have recurrent attacks (crises) of fever and pain, most often in the abdomen, joints, and extremities. These attacks are attributed to vascular occlusion when red blood cells (RBCs) assume the characteristic sickled shape. Crises are usually triggered by dehydration, hypoxia, or acidosis. Women with the sickle cell trait are usually at a greater risk for postpartum endometritis (uterine wall infection); however, this development is not likely to occur during the pregnancy and is not a sign for the disorder. Although women with sickle cell anemia are at an increased risk for UTIs, these infections are not an indication of a sickle cell crisis.

Which medications can be taken by postmenopausal women to treat and/or prevent osteoporosis? (Select all that apply.) a. Calcium b. NSAIDs c. Alendronate d. Risedronate sodium e. Calcitonin

A, C, D, E Calcium, raloxifene, alendronate, risedronate sodium, and Calcitonin can be used by postmenopausal women to treat or prevent osteoporosis. Parathyroid hormone and estrogen may also be of value. NSAIDs may provide pain relief; however, these medications neither prevent nor treat osteoporosis.

A client who has undergone a dilation and curettage (D&C)for early pregnancy loss is likely to be discharged the same day. The nurse must ensure that her vital signs are stable, that bleeding has been controlled, and that the woman has adequately recovered from the administration of anesthesia. To promote an optimal recovery, what information should discharge teaching include? (Select all that apply.) a. Iron supplementation b. Resumption of intercourse at 6 weeks post-procedure c. Referral to a support group, if necessary d. Expectation of heavy bleeding for at least 2 weeks e. Emphasizing the need for rest

A, C, E The woman should be advised to consume a diet high in iron and protein. For many women, iron supplementation also is necessary. The nurse should acknowledge that the client has experienced a loss, however early. She can be taught to expect mood swings and possibly depression. Referral to a support group, clergy, or professional counseling may be necessary. Discharge teaching should emphasize the need for rest. Nothing should be placed in the vagina for 2 weeks after the procedure, including tampons and vaginal intercourse. The purpose of this recommendation is to prevent infection. Should infection occur, antibiotics may be prescribed. The client should expect a scant, dark discharge for 1 to 2 weeks. Should heavy, profuse, or bright bleeding occur, she should be instructed to contact her health care provider.

Which adverse prenatal outcomes are associated with the HELLP syndrome? (Select all that apply.) a. Placental abruption b. Placenta previa c. Renal failure d. Cirrhosis e. Maternal and fetal death

A, C, E The HELLP syndrome is associated with an increased risk for adverse perinatal outcomes, including placental abruption, acute renal failure, subcapsular hepatic hematoma, hepatic rupture, recurrent preeclampsia, preterm birth, and fetal and maternal death. The HELLP syndrome is associated with an increased risk for placental abruption, not placenta previa. It is also associated with an increased risk for hepatic hematoma, not cirrhosis.

The nurse is reviewing the educational packet provided to a client about tubal ligation. Which information regarding this procedure is important for the nurse to share? (Select all that apply.) a. "It is highly unlikely that you will become pregnant after the procedure." b. "Tubal ligation is an effective form of 100% permanent sterilization. You won't be able to get pregnant." c. "Sterilization offers some form of protection against sexually transmitted infections (STIs)." d. "Sterilization offers no protection against sexually transmitted infections (STIs)." e. "Your menstrual cycle will greatly increase after your sterilization."

A, D A woman is unlikely to become pregnant after tubal ligation. However, sterilization offers no protection against STIs and is not 100% effective. Typically, the menstrual cycle remains the same after a tubal ligation.

A client has requested information regarding alternatives to hormonal therapy for menopausal symptoms. Which current information should the nurse provide to the client? (Select all that apply.) a. Soy b. Vitamin C c. Vitamin K d. Vitamin E e. Vitamin A

A, D Both soy and vitamin E have been reported to help alleviate menopausal symptoms, and both are readily available in food sources. Vitamin E can be also be taken as a supplement. Vitamins C, K, and A have no apparent effect on menopausal symptoms.

Nurses should be cognizant of what information regarding the non-contraceptive medical effects of combination oral contraceptives (COCs)? a. COCs can cause TSS if the prescription is wrong. b. Hormonal withdrawal bleeding is usually a little more profuse than in normal menstruation and lasts a week for those who use COCs. c. COCs increase the risk of endometrial and ovarian cancers. d. Effectiveness of COCs can be altered by some over-the-counter medications and herbal supplements.

d. Effectiveness of COCs can be altered by some over-the-counter medications and herbal supplements. The effectiveness of COCs can be altered by some over-the-counter medications and herbal supplements. TSS can occur in some who use the diaphragm, but it is not a consequence of taking oral contraceptive pills. Hormonal withdrawal bleeding usually is lighter than in normal menstruation and lasts a couple of days. Oral contraceptive pills offer protection against the risk of endometrial and ovarian cancers.

Postoperative care of the pregnant woman who requires abdominal surgery for appendicitis includes which additional assessment? a. Intake and output (I&O) and intravenous (IV) site b. Signs and symptoms of infection c. Vital signs and incision d. Fetal heart rate (FHR) and uterine activity

d. Fetal heart rate (FHR) and uterine activity Care of a pregnant woman undergoing surgery for appendicitis differs from that for a nonpregnant woman in one significant aspect: the presence of the fetus. Continuous fetal and uterine monitoring should take place. An assessment of I&O levels, along with an assessment of the IV site, are normal postoperative care procedures. Evaluating the client for signs and symptoms of infection is also part of routine postoperative care. Routine vital signs and evaluation of the incision site are expected components of postoperative care.

A 32-year-old primigravida is admitted with a diagnosis of ectopic pregnancy. Which information assists the nurse in developing the plan of care? a. Bed rest and analgesics are the recommended treatment. b. She will be unable to conceive in the future. c. A D&C will be performed to remove the products of conception d. Hemorrhage is the primary concern.

d. Hemorrhage is the primary concern. Severe bleeding occurs if the fallopian tube ruptures. The recommended treatment is to remove the pregnancy before rupture to prevent hemorrhaging. If the tube must be removed, then the woman's fertility will decrease; however, she will not be infertile. A D&C is performed on the inside of the uterine cavity. The ectopic pregnancy is located within the tubes.

Which statement regarding the condition referred to as a miscarriage is most accurate? a. A miscarriage is a natural pregnancy loss before labor begins. b. It occurs in fewer than 5% of all clinically recognized pregnancies. c. Careless maternal behavior, such as poor nutrition or excessive exercise, can be a factor in causing a miscarriage. d. If a miscarriage occurs before the 12th week of pregnancy, then it may be observed only as moderate discomfort and blood loss

d. If a miscarriage occurs before the 12th week of pregnancy, then it may be observed only as moderate discomfort and blood loss Before the sixth week, the only evidence might be a heavy menstrual flow. After the 12th week, more severe pain, like that of labor, is likely. Miscarriage is a natural pregnancy loss, but it occurs, by definition, before 20 weeks of gestation, before the fetus is viable. Miscarriages occur in approximately 10% to 15% of all clinically recognized pregnancies. Miscarriages can be caused by several disorders or illnesses outside the mother's control or knowledge.

The client is being induced in response to worsening preeclampsia. She is also receiving magnesium sulfate. It appears that her labor has not become active, despite several hours of oxytocin administration. She asks the nurse, "Why is this taking so long?" What is the nurse's most appropriate response? a. "Since the magnesium is competing with the oxytocin, your labor is slowed." b. "I don't know why it is taking so long." c. "The length of labor varies for different women." d. "Your baby is just being stubborn."

a. "Since the magnesium is competing with the oxytocin, your labor is slowed." Because magnesium sulfate is a tocolytic agent, its use may increase the duration of labor. The amount of oxytocin needed to stimulate labor may be more than that needed for the woman who is not receiving magnesium sulfate. The nurse should explain to the client the effects of magnesium sulfate on the duration of labor. Although the length of labor varies for different women, the most likely reason this woman's labor is protracted is the tocolytic effects of magnesium sulfate. The behavior of the fetus has no bearing on the length of labor.

Which statement regarding abnormal uterine bleeding (AUB) is most accurate? a. AUB is most commonly caused by anovulation. b. AUB most often occurs in middle age. c. The diagnosis of AUB should be the first consideration for abnormal menstrual bleeding. d. Steroids are the most effective medical treatment for AUB.

a. AUB is most commonly caused by anovulation. Anovulation may occur because of hypothalamic dysfunction or polycystic ovary syndrome. AUB most often occurs when the menstrual cycle is being established or when it draws to a close at menopause. A diagnosis of AUB is made only after all other causes of abnormal menstrual bleeding have been ruled out. The most effective medical treatment is oral or intravenous estrogen.

In caring for the woman with disseminated intravascular coagulation (DIC), which order should the nurse anticipate? a. Administration of blood b. Preparation of the client for invasive hemodynamic monitoring c. Restriction of intravascular fluids d. Administration of steroids

a. Administration of blood Primary medical management in all cases of DIC involves a correction of the underlying cause, volume replacement, blood component therapy, optimization of oxygenation and perfusion status, and continued reassessment of laboratory parameters. Central monitoring would not be initially ordered in a client with DIC because it could contribute to more areas of bleeding. Management of DIC would include volume replacement, not volume restriction. Steroids are not indicated for the management of DIC.

The female athlete triad includes which common menstrual disorder? a. Amenorrhea b. Dysmenorrhea c. Menorrhagia d. Metrorrhagia

a. Amenorrhea The interrelatedness of disordered eating, amenorrhea, and altered bone mineral density have been described as the female athlete triad. Dysmenorrhea is painful menstruation that begins 2 to 6 months after menarche. Menorrhagia is abnormally profuse or excessive bleeding from the uterus. Metrorrhagia is bleeding between periods and can be caused by progestin injections and implants.

What is the highest priority nursing intervention when admitting a pregnant woman who has experienced a bleeding episode in late pregnancy? a. Assessing fetal heart rate (FHR) and maternal vital signs b. Performing a venipuncture for hemoglobin and hematocrit levels c. Placing clean disposable pads to collect any drainage d. Monitoring uterine contractions

a. Assessing fetal heart rate (FHR) and maternal vital signs Assessment of the FHR and maternal vital signs will assist the nurse in determining the degree of the blood loss and its effect on the mother and fetus. The most important assessment is to check the well-being of both the mother and the fetus. The blood levels can be obtained later. Assessing future bleeding is important; however, the top priority remains mother/fetal well-being. Monitoring uterine contractions is important but not a top priority.

Which order should the nurse expect for a client admitted with a threatened abortion? a. Bed rest b. Administration of ritodrine IV c. Nothing by mouth (nil per os [NPO]) d. Narcotic analgesia every 3 hours, as needed

a. Bed rest Decreasing the woman's activity level may alleviate the bleeding and allow the pregnancy to continue. Ritodrine is not the first drug of choice for tocolytic medications. Having the woman placed on NPO is unnecessary. At times, dehydration may produce contractions; therefore, hydration is important. Narcotic analgesia will not decrease the contractions and may mask the severity of the contractions.

A perimenopausal client has arrived for her annual gynecologic examination. Which preexisting condition would be extremely important for the nurse to identify during a discussion regarding the risks and benefits of hormone therapy? a. Breast cancer b. Vaginal and urinary tract atrophy c. Osteoporosis d. Arteriosclerosis

a. Breast cancer Women with a high risk for breast cancer should be counseled against using estrogen replacement therapy (ERT). Estrogen prevents the atrophy of vaginal and urinary tract tissue and protects against the development of osteoporosis. Estrogen also has a favorable effect on circulating lipids, reducing low-density lipoprotein (LDL) and total cholesterol levels and increasing high-density lipoprotein (HDL) levels. It also has a direct antiatherosclerotic effect on the arteries.

Which statement concerning the complication of maternal diabetes is the most accurate? a. Diabetic ketoacidosis (DKA) can lead to fetal death at any time during pregnancy. b. Hydramnios occurs approximately twice as often in diabetic pregnancies than in nondiabetic pregnancies. c. Infections occur about as often and are considered about as serious in both diabetic and nondiabetic pregnancies. d. Even mild-to-moderate hypoglycemic episodes can have significant effects on fetal well-being.

a. Diabetic ketoacidosis (DKA) can lead to fetal death at any time during pregnancy. Prompt treatment of DKA is necessary to save the fetus and the mother. Hydramnios occurs 10 times more often in diabetic pregnancies. Infections are more common and more serious in pregnant women with diabetes. Mild-to-moderate hypoglycemic episodes do not appear to have significant effects on fetal well-being.

Which is the most common technique used for the termination of a pregnancy in the second trimester? a. Dilation and evacuation (D&E) b. Methotrexate administration c. Prostaglandin administration d. Vacuum aspiration

a. Dilation and evacuation (D&E) D&E can be performed at any point up to 20 weeks of gestation. It is more commonly performed between 13 and 16 weeks of gestation. Methotrexate is a cytotoxic drug that causes early abortion by preventing fetal cell division. Prostaglandins are also used for early abortion and work by dilating the cervix and initiating uterine wall contractions. Vacuum aspiration is used for abortions in the first trimester.

In caring for an immediate postpartum client, the nurse notes petechiae and oozing from her intravenous (IV) site. The client would be closely monitored for which clotting disorder? a. Disseminated intravascular coagulation (DIC) b. Amniotic fluid embolism (AFE) c. Hemorrhage d. HELLP syndrome

a. Disseminated intravascular coagulation (DIC) The diagnosis of DIC is made according to clinical findings and laboratory markers. A physical examination reveals unusual bleeding. Petechiae may appear around a blood pressure cuff on the woman's arm. Excessive bleeding may occur from the site of slight trauma such as venipuncture sites. These symptoms are not associated with AFE, nor is AFE a bleeding disorder. Hemorrhage occurs for a variety of reasons in the postpartum client. These symptoms are associated with DIC. Hemorrhage would be a finding associated with DIC and is not a clotting disorder in and of itself. HELLP syndrome is not a clotting disorder, but it may contribute to the clotting disorder DIC.

Which statement regarding emergency contraception is correct? a. Emergency contraception requires that the first dose be taken within 120 hours of unprotected intercourse. b. Emergency contraception may be taken right after ovulation. c. Emergency contraception has an effectiveness rate in preventing pregnancy of approximately 50%. d. Emergency contraception is commonly associated with the side effect of menorrhagia.

a. Emergency contraception requires that the first dose be taken within 120 hours of unprotected intercourse. Emergency contraception should be taken as soon as possible or within 72 hours of unprotected intercourse to prevent pregnancy. If taken before ovulation, follicular development is inhibited, which prevents ovulation. The risk of pregnancy is reduced by as much as 75%. The most common side effect of postcoital contraception is nausea.

A client reports experiencing severe abdominal and pelvic pain around the time of menstruation. This pain has become progressively worse over the last 5 years. She also experiences pain during intercourse and has tried unsuccessfully to become pregnant for the past 18 months. To which condition are these symptoms most likely related? a. Endometriosis b. Premenstrual Syndrome (PMS) c. Primary dysmenorrhea d. Secondary dysmenorrhea

a. Endometriosis Symptoms of endometriosis can change over time and may not reflect the extent of the disease. Major symptoms include dysmenorrhea and deep pelvic dyspareunia (painful intercourse). Impaired fertility may result from adhesions caused by endometriosis. Although endometriosis may be associated with secondary dysmenorrhea, it is not a cause of primary dysmenorrhea or PMS. In addition, this woman is reporting symptoms of dyspareunia and infertility, which are associated with endometriosis, not with PMS or primary or secondary dysmenorrhea.

Which term best describes the conscious decision concerning deciding when to conceive or avoid pregnancy as opposed to the intentional prevention of pregnancy during intercourse? a. Family planning b. Birth control c. Contraception d. Assisted reproductive therapy

a. Family planning Family planning is the process of deciding when and if to have children. Birth control is the device and/or practice used to reduce the risk of conceiving or bearing children. Contraception is the intentional prevention of pregnancy during sexual intercourse. Assisted reproductive therapy is one of several possible treatments for infertility.

A woman experiencing severe preeclampsia has been receiving magnesium sulfate by intravenous infusion for 8 hours. The nurse assesses the client and documents the following findings: temperature of 37.1° C, pulse rate of 96 beats per minute, respiratory rate of 24 breaths per minute, BP of 155/112 mm Hg, 3+ DTRs, and no ankle clonus. The nurse calls the provider with an update. The nurse should anticipate an order for which medication? a. Hydralazine b. Magnesium sulfate bolus c. Diazepam d. Calcium gluconate

a. Hydralazine Hydralazine is an antihypertensive medication commonly used to treat hypertension in severe preeclampsia. Typically, it is administered for a systolic BP higher than 160 mm Hg or a diastolic BP higher than 110 mm Hg. An additional bolus of magnesium sulfate may be ordered for increasing signs of CNS irritability related to severe preeclampsia (e.g., clonus) or if eclampsia develops. Diazepam is sometimes used to stop or shorten eclamptic seizures. Calcium gluconate is used as the antidote for magnesium sulfate toxicity. The client is not currently displaying any signs or symptoms of magnesium toxicity.

What is the most common medical complication of pregnancy? a. Hypertension b. Hyperemesis gravidarum c. Hemorrhagic complications d. Infections

a. Hypertension Preeclampsia and eclampsia are two noted deadly forms of hypertension. A large percentage of pregnant women will have nausea and vomiting, but a relatively few will have the severe form called hyperemesis gravidarum. Hemorrhagic complications are the second most common medical complication of pregnancy; hypertension is the most common. Infection is a risk factor for preeclampsia.

Which major neonatal complication is carefully monitored after the birth of the infant of a diabetic mother? a. Hypoglycemia b. Hypercalcemia c. Hypobilirubinemia d. Hypoinsulinemia

a. Hypoglycemia The neonate is at highest risk for hypoglycemia because fetal insulin production is accelerated during pregnancy to metabolize excessive glucose from the mother. At birth, the maternal glucose supply stops and the neonatal insulin exceeds the available glucose, thus leading to hypoglycemia. Hypocalcemia is associated with preterm birth, birth trauma, and asphyxia, all common problems of the infant of a diabetic mother. Excess erythrocytes are broken down after birth, and large amounts of bilirubin are released into the neonate's circulation, with resulting hyperbilirubinemia. Because fetal insulin production is accelerated during pregnancy, hyperinsulinemia develops in the neonate.

Which intervention is most important when planning care for a client with severe gestational hypertension? a. Induction of labor is likely, as near term as possible. b. If at home, the woman should be confined to her bed, even with mild gestational hypertension. c. Special diet low in protein and salt should be initiated immediately. d. Vaginal birth is still an option, even in severe cases.

a. Induction of labor is likely, as near term as possible. By 34 weeks of gestation, the risk of continuing the pregnancy may be considered greater than the risks of a preterm birth. Strict bed rest is controversial for mild cases; some women in the hospital are even allowed to move around. Diet and fluid recommendations are essentially the same as for healthy pregnant women, although some authorities have suggested a diet high in protein. Women with severe gestational hypertension should expect a cesarean delivery.

Which neonatal complications are associated with hypertension in the mother? a. Intrauterine growth restriction (IUGR) and prematurity b. Seizures and cerebral hemorrhage c. Hepatic or renal dysfunction d. Placental abruption and DIC

a. Intrauterine growth restriction (IUGR) and prematurity Neonatal complications are related to placental insufficiency and include IUGR, prematurity, and necrotizing enterocolitis. Seizures and cerebral hemorrhage are maternal complications. Hepatic and renal dysfunction are maternal complications of hypertensive disorders in pregnancy. Placental abruption and DIC are conditions related to maternal morbidity and mortality.

Screening at 24 weeks of gestation reveals that a pregnant woman is experiencing gestational diabetes mellitus (GDM). In planning her care, the nurse and the client mutually agree that an expected outcome is to prevent injury to the fetus because of GDM. This fetus is at the greatest risk for which condition? a. Macrosomia b. Congenital anomalies of the central nervous system c. Preterm birth d. Low birth weight

a. Macrosomia Poor glycemic control later in pregnancy increases the rate of fetal macrosomia. Poor glycemic control during the preconception time frame and into the early weeks of the pregnancy is associated with congenital anomalies. Preterm labor or birth is more likely to occur with severe diabetes and is the greatest risk in women with pregestational diabetes. Increased weight, or macrosomia, is the greatest risk factor for this fetus.

With one exception, the safest pregnancy is one during which the woman is drug and alcohol free. What is the optimal treatment for women addicted to opioids? a. Methadone b. Detoxification c. Smoking cessation d. 4 Ps Plus

a. Methadone Methadone is currently considered the standard of care for pregnant women who are dependent on heroin or other narcotics. Buprenorphine is another medication approved for the treatment of opioid addiction that is increasingly being used during pregnancy. Opioid replacement therapy has been shown to decrease opioid and other drug use, reduce criminal activity, improve individual functioning, and decrease the rates of infections such as hepatitis B and C, human immunodeficiency virus (HIV), and other STIs. Detoxification is the treatment used for alcohol addiction. Pregnant women requiring withdrawal from alcohol should be admitted for inpatient management. Women are more likely to stop smoking during pregnancy than at any other time in their lives. A smoking cessation program can assist in achieving this goal. The 4 Ps Plus is a screening tool specifically designed to identify pregnant women who need in-depth assessment related to substance abuse.

A woman has chosen the calendar method of conception control. Which is the most important action the nurse should perform during the assessment process in preparation to discuss the implementation of this method? a. Obtain a history of the woman's menstrual cycle lengths for the past 6 to 12 months. b. Determine the client's weight gain and loss pattern for the previous year. c. Examine skin pigmentation and hair texture for hormonal changes. d. Explore the client's previous experiences with conception control.

a. Obtain a history of the woman's menstrual cycle lengths for the past 6 to 12 months. The calendar method of conception control is based on the number of days in each cycle, counting from the first day of menses. The fertile period is determined after the lengths of menstrual cycles have been accurately recorded for 6 months. Weight gain or loss may be partly related to hormonal fluctuations, but it has no bearing on the use of the calendar method. Integumentary changes may be related to hormonal changes, but they are not indicators for use of the calendar method. Exploring previous experiences with conception control may demonstrate client understanding and compliancy, but these experiences are not the most important aspect to assess for the discussion of the calendar method.

What nursing diagnosis is the most appropriate for a woman experiencing severe preeclampsia? a. Potential for injury to mother and fetus, related to central nervous system (CNS) irritability b. Potential for reduced gas exchange c. Potential for inadequate fluid volume, related to increased sodium retention secondary to the administration of magnesium sulfate d. Potential for increased cardiac output, related to the use of antihypertensive drugs

a. Potential for injury to mother and fetus, related to central nervous system (CNS) irritability Potential for injury is the most appropriate nursing diagnosis for this client scenario. Gas exchange is more likely to become reduced, attributable to pulmonary edema. A potential for increased, not decreased, fluid volume, related to increased sodium retention, and a potential for decreased, not increased, cardiac output, related to the use of antihypertensive drugs, also is increased.

What is the correct definition of a spontaneous termination of a pregnancy (abortion)? a. Pregnancy is less than 20 weeks. b. Fetus weighs less than 1000 g. c. Products of conception are passed intact. d. No evidence exists of intrauterine infection.

a. Pregnancy is less than 20 weeks. An abortion is the termination of pregnancy before the age of viability (20 weeks). The weight of the fetus is not considered because some older fetuses may have a low birth weight. A spontaneous abortion may be complete or incomplete and may be caused by many problems, one being intrauterine infection.

According to research, which risk factor for postpartum depression (PPD) is likely to have the greatest effect on the client postpartum? a. Prenatal depression b. Single-mother status c. Low socioeconomic status d. Unplanned or unwanted pregnancy

a. Prenatal depression Prenatal depression has been found to be a major risk factor for PPD. Single-mother status and low socioeconomic status are both small-relationship predictors for PPD. Although an unwanted pregnancy may contribute to the risk for PPD, it does not pose as great an effect as prenatal depression.

A 21-year-old client reports experiencing severe pain immediately after the commencement of her menses. Which gynecologic condition is the most likely cause of this client's presenting symptoms? a. Primary dysmenorrhea b. Secondary dysmenorrhea c. Dyspareunia d. Endometriosis

a. Primary dysmenorrhea Primary dysmenorrhea, or pain during or shortly before menstruation, has a biochemical basis and arises from the release of prostaglandins with menses. Secondary dysmenorrhea develops after the age of 25 years and is usually associated with a pelvic pathologic condition. Dyspareunia, or painful intercourse, is commonly associated with endometriosis. Endometriosis is characterized by endometrial glands and stoma outside of the uterus.

Despite warnings, prenatal exposure to alcohol continues to far exceed exposure to illicit drugs. Which condition is rarely associated with fetal alcohol syndrome (FAS)? a. Respiratory conditions b. Intellectual impairment c. Neural development disorder d. Alcohol-related birth defects (ARBDs)

a. Respiratory conditions Respiratory difficulties are not attributed to exposure to alcohol in utero. Other abnormalities related to FAS include mental retardation, neurodevelopment disorders, and ARBDs.

While interviewing a 31-year-old woman before her routine gynecologic examination, the nurse collects data about the client's recent menstrual cycles. Which statement by the client should prompt the nurse to collect further information? a. "My menstrual flow lasts 5 to 6 days." b. "My flow is very heavy." c. "I have had a small amount of spotting midway between my periods for the past 2 months." d. "The length of my menstrual cycles varies from 26 to 29 days."

b. "My flow is very heavy." Menorrhagia is defined as excessive menstrual bleeding, either in duration or in amount. Heavy bleeding can have many causes. The amount of bleeding and its effect on daily activities should be evaluated. A menstrual flow that lasts 5 to 6 days is a normal finding. Mittlestaining, a small amount of bleeding or spotting that occurs at the time of ovulation (14 days before the onset of the next menses), is considered normal. During her reproductive years, a woman may have physiologic variations in her menstrual cycle. Variations in the length of a menstrual cycle are considered normal.

A male client asks the nurse why it is better to purchase condoms that are not lubricated with nonoxynol-9 (a common spermicide). Which response by the nurse is the most accurate? a. "The lubricant prevents vaginal irritation." b. "Nonoxynol-9 does not provide protection against STIs as originally thought" c. "The additional lubrication improves sex." d. "Nonoxynol-9 improves penile sensitivity."

b. "Nonoxynol-9 does not provide protection against STIs as originally thought" Nonoxynol-9 does not provide protection against STIs as originally thought; it has also been linked to an increase in the transmission of the HIV and can cause genital lesions. Nonoxynol-9 may cause vaginal irritation, has no effect on the quality of sexual activity, and has no effect on penile sensitivity.

A perinatal nurse is giving discharge instructions to a woman, status post-suction, and curettage secondary to a hydatidiform mole. The woman asks why she must take oral contraceptives for the next 12 months. What is the best response by the nurse? a. "If you get pregnant within 1 year, the chance of a successful pregnancy is very small. Therefore, if you desire a future pregnancy, it would be better for you to use the most reliable method of contraception available." b. "The major risk to you after a molar pregnancy is a type of cancer that can be diagnosed only by measuring the same hormone that your body produces during pregnancy. If you were to get pregnant, then it would make the diagnosis of this cancer more difficult." c. "If you can avoid a pregnancy for the next year, the chance of developing a second molar pregnancy is rare. Therefore, to improve your chance of a successful pregnancy, not getting pregnant at this time is best." d. "Oral contraceptives are the only form of birth control that will prevent a recurrence of a molar pregnancy."

b. "The major risk to you after a molar pregnancy is a type of cancer that can be diagnosed only by measuring the same hormone that your body produces during pregnancy. If you were to get pregnant, then it would make the diagnosis of this cancer more difficult." Beta-human chorionic gonadotropin (beta-hCG) hormone levels are drawn for 1 year to ensure that the mole is completely gone. The chance of developing choriocarcinoma after the development of a hydatidiform mole is increased. Therefore, the goal is to achieve a zero human chorionic gonadotropin (hCG) level. If the woman were to become pregnant, then it may obscure the presence of the potentially carcinogenic cells. Women should be instructed to use birth control for 1 year after treatment for a hydatidiform mole. The rationale for avoiding pregnancy for 1 year is to ensure that carcinogenic cells are not present. Any contraceptive method except an intrauterine device (IUD) is acceptable.

A woman who has a seizure disorder and takes barbiturates and phenytoin sodium daily asks the nurse about the pill as a contraceptive choice. What is the nurse's best response? a. "Oral contraceptives are a highly effective method, but they have some side effects." b. "Your current medications will reduce the effectiveness of the pill." c. "Oral contraceptives will reduce the effectiveness of your seizure medication." d. "The pill is a good choice for a woman of your age and with your personal history."

b. "Your current medications will reduce the effectiveness of the pill." Because the liver metabolizes oral contraceptives, their effectiveness is reduced when they are simultaneously taken with anticonvulsants. Stating that the pill is an effective birth control method with side effects is a true statement, but this response is not the most appropriate. The anticonvulsant reduces the effectiveness of the pill, not the other way around. Stating that the pill is a good choice for a woman of her age and personal history does not teach the client that the effectiveness of the pill may be reduced because of her anticonvulsant therapy.

Which contraceptive method should the nurse identify as protecting against sexually transmitted infections (STIs) and the human immunodeficiency virus (HIV)? a. Periodic abstinence b. Barrier methods c. Hormonal methods d. Same protection with all methods

b. Barrier methods Barrier methods, such as condoms, protect against STIs and the HIV the best of all contraceptive methods. Periodic abstinence and hormonal methods, such as birth control pills, offer no protection against STIs or the HIV.

Which maternal condition always necessitates delivery by cesarean birth? a. Marginal placenta previa b. Complete placenta previa c. Ectopic pregnancy d. Eclampsia

b. Complete placenta previa In complete placenta previa, the placenta completely covers the cervical os. A cesarean birth is the acceptable method of delivery. The risk of fetal death occurring is due to preterm birth. If the previa is marginal (i.e., 2 cm or greater away from the cervical os), then labor can be attempted. A cesarean birth is not indicated for an ectopic pregnancy. Labor can be safely induced if the eclampsia is under control.

Importantly, the nurse must be aware of which information related to the use of intrauterine devices (IUDs)? a. Return to fertility can take several weeks after the device is removed. b. Copper IUDs can serve as an emergency contraception under certain situations. c. IUDs offer the same protection against STIs as the diaphragm. d. Consent forms are not needed for IUD insertion.

b. Copper IUDs can serve as an emergency contraception under certain situations. The woman has up to 5 days to insert the IUD after unprotected sex. The return to fertility is immediate after the removal of the IUD. IUDs offer no protection against STIs. A consent form is required for insertion, as is a negative pregnancy test.

Which action should the nurse first take when meeting with a new client to discuss contraception? a. Obtain data about the frequency of coitus. b. Determine the woman's level of knowledge concerning contraception. c. Assess the woman's willingness to touch her genitals and cervical mucus. d. Evaluate the woman's contraceptive life plan.

b. Determine the woman's level of knowledge concerning contraception. Determining the woman's level of knowledge concerning contraception and her commitment to any particular method is the primary step of this nursing assessment and necessary before completing the process and moving on to a nursing diagnosis. Once the client's level of knowledge is determined, the nurse can interact with the woman to compare options, reliability, cost, comfort level, protection from STIs, and her partner's willingness to participate. Although important, obtaining data about the frequency of coitus is not the first action that the nurse should undertake when completing an assessment. Data should include not only the frequency of coitus but also the number of sexual partners, level of contraceptive involvement, and the partner's objections. Assessing the woman's willingness to touch herself is a key factor for the nurse to discuss should the client express an interest in using one of the fertility awareness methods of contraception. The nurse must be aware of the client's plan regarding whether she is attempting to prevent conception, delay conception, or conceive.

The client makes an appointment for preconception counseling. The woman has a known heart condition and is unsure if she should become pregnant. Which is the only cardiac condition that would cause concern? a. Marfan syndrome b. Eisenmenger syndrome c. Heart transplant d. Ventricular septal defect (VSD)

b. Eisenmenger syndrome Pregnancy is contraindicated in clients with Eisenmenger syndrome. Women who have had heart transplants are successfully having babies. However, conception should be postponed for at least 1 year after transplantation. Management of the client with Marfan syndrome during pregnancy includes bed rest, beta-blockers, and surgery before conception. VSD is usually corrected early in life and is therefore not a contraindication to pregnancy.

To assist a client in managing the symptoms of Premenstrual Syndrome (PMS), what intervention should the nurse recommend based on current evidence? a. Adhering to a diet with more body-building and energy foods, such as carbohydrates b. Engaging in herbal therapies, yoga, and massage therapy c. Seeking antidepressant therapy for symptom control d. Avoiding the use of natural diuretics

b. Engaging in herbal therapies, yoga, and massage therapy Herbal therapies, yoga, and massage have been reported to have a beneficial effect on the symptoms of PMS. Limiting red meat, simple carbohydrates, caffeinated beverages, and alcohol improves the diet and may mitigate symptoms. Medication is usually begun only if lifestyle changes fail to provide significant relief. Natural diuretics may help reduce fluid retention.

It is extremely rare for a woman to die in childbirth; however, it can happen. In the United States, the annual occurrence of maternal death is 12 per 100,000 cases of live birth. What are the leading causes of maternal death? a. Embolism and preeclampsia b. Falls and motor vehicle accidents (MVAs) c. Hemorrhage and infection d. Underlying chronic conditions

b. Falls and motor vehicle accidents (MVAs) Trauma is the leading cause of obstetric death in women of childbearing age. Most maternal injuries are the result of MVAs and falls. Although preeclampsia and embolism are significant contributors to perinatal morbidity, these are not the leading cause of maternal mortality. Maternal death caused by trauma may occur as the result of hemorrhagic shock or abruptio placentae. In these cases, the hemorrhage is the result of trauma, not childbirth. The wish to become a parent is not eliminated by a chronic health problem, and many women each year risk their lives to have a baby. Because of advanced pediatric care, many women are surviving childhood illnesses and reaching adulthood with chronic health problems such as cystic fibrosis, diabetes, and pulmonary disorders.

A 26-year-old primigravida has come to the clinic for her regular prenatal visit at 12 weeks. She appears thin and somewhat nervous. She reports that she eats a well-balanced diet, although her weight is 5 pounds less than it was at her last visit. The results of laboratory studies confirm that she has a hyperthyroid condition. Based on the available data, the nurse formulates a plan of care. Which nursing diagnosis is most appropriate for the client currently? a. Disrupted fluid balance b. Inadequate nutrition c. Excessive nutrition d. Disrupted sleep

b. Inadequate nutrition This client's clinical cues include weight loss, which supports a nursing diagnosis of "Inadequate nutrition." No clinical signs or symptoms support a nursing diagnosis of disrupted fluid balance. This client reports weight loss not weight gain. Although the client reports nervousness, the most appropriate nursing diagnosis, based on the client's other clinical symptoms, is "Inadequate nutrition."

A client with a history of bipolar disorder is called by the postpartum support nurse for follow-up. Which symptoms would reassure the nurse that the client is not experiencing a manic episode? a. Psychomotor agitation and rapid speech patterns b. Increased appetite and sleeping 8 hours a night c. Hyperactivity and distractibility d. Pressured speech and grandiosity

b. Increased appetite and sleeping 8 hours a night An increased appetite and a lack of interest would reassure the nurse that the client is not experiencing an episode of mania. Clinical manifestations of a manic episode include at least three of the following: grandiosity, decreased need for sleep, pressured speech, flight of ideas, distractibility, psychomotor agitation, and excessive involvement in pleasurable activities. The pregnant woman exhibiting symptoms of a manic episode will likely have a decreased interest in eating and an increased level of interest in pleasurable activities without regard for negative consequences. Psychomotor agitation and a lack of sleep, hyperactivity and distractibility, and pressured speech and grandiosity are all clinical manifestations of a manic episode.

In contrast to placenta previa, what is the most prevalent clinical manifestation of abruptio placentae? a. Bleeding b. Intense abdominal pain c. Uterine activity d. Cramping

b. Intense abdominal pain Pain is absent with placenta previa and may be agonizing with abruptio placentae. Bleeding may be present in varying degrees for both placental conditions. Uterine activity and cramping may be present with both placental conditions.

The nurse who is caring for a woman hospitalized for hyperemesis gravidarum would expect the initial treatment to involve what? a. Corticosteroids to reduce inflammation b. Intravenous (IV) therapy to correct fluid and electrolyte imbalances c. Antiemetic medication, such as pyridoxine, to control nausea and vomiting d. Enteral nutrition to correct nutritional deficits

b. Intravenous (IV) therapy to correct fluid and electrolyte imbalances Initially, the woman who is unable to down clear liquids by mouth requires IV therapy to correct fluid and electrolyte imbalances. Corticosteroids have been successfully used to treat refractory hyperemesis gravidarum, but they are not the expected initial treatment for this disorder. Pyridoxine is vitamin B6, not an antiemetic medication. Promethazine, a common antiemetic, may be prescribed. In severe cases of hyperemesis gravidarum, enteral nutrition via a feeding tube may be necessary to correct maternal nutritional deprivation but is not the initial treatment for this client.

An 18-year-old client who has reached 16 weeks of gestation was recently diagnosed with pre-gestational diabetes. She attends her centering appointment accompanied by one of her girlfriends. This young woman appears more concerned about how her pregnancy will affect her social life than her recent diagnosis of diabetes. Several nursing diagnoses are applicable to assist in planning adequate care. What is the most appropriate diagnosis at this time? a. Potential for injury to the fetus related to birth trauma b. Lack of understanding related to diabetic pregnancy management c. Lack of understanding related to insulin administration d. Potential for injury to the mother related to hypoglycemia or hyperglycemia

b. Lack of understanding related to diabetic pregnancy management Before a treatment plan is developed or goals for the outcome of care are outlined, this client must come to an understanding of diabetes and the potential effects on her pregnancy. She appears more concerned about changes to her social life than adopting a new self-care regimen. Potential for injury to the fetus related to either placental insufficiency or birth trauma may come later in the pregnancy. At this time, the client is having difficulty acknowledging the adjustments that she needs to make to her lifestyle to care for herself during pregnancy. The client may not yet be on insulin. Insulin requirements increase with gestation. The importance of glycemic control must be part of health teaching for this client. However, she has not yet acknowledged that changes to her lifestyle need to be made and may not participate in the plan of care until understanding takes place.

The use of methamphetamine (meth) has been described as a significant drug problem in the United States. The nurse who provides care to this client population should be cognizant of what regarding methamphetamine use? a. Methamphetamines are similar to opiates. b. Methamphetamines are stimulants with vasoconstrictive characteristics. c. Methamphetamines should not be discontinued during pregnancy. d. Methamphetamines are associated with a low rate of relapse.

b. Methamphetamines are stimulants with vasoconstrictive characteristics. Methamphetamines are stimulants with vasoconstrictive characteristics similar to cocaine and are similarly used. As is the case with cocaine users, methamphetamine users are urged to immediately stop all use during pregnancy. Unfortunately, because methamphetamine users are extremely psychologically addicted, the rate of relapse is extremely high.

Preconception counseling is critical in the safe management of diabetic pregnancies. Which complication is commonly associated with poor glycemic control before and during early pregnancy? a. Frequent episodes of maternal hypoglycemia b. Miscarriage c. Hydramnios d. Hyperemesis gravidarum

b. Miscarriage Preconception counseling is particularly important since strict metabolic control before conception and in the early weeks of gestation is instrumental in decreasing the risk of miscarriages. Frequent episodes of maternal hypoglycemia may occur during the first trimester (not before conception) as a result of hormonal changes and the effects on insulin production and use. Hydramnios occurs approximately 10 times more often in diabetic pregnancies than in nondiabetic pregnancies. Typically, it is observed in the third trimester of pregnancy. Hyperemesis gravidarum may exacerbate hypoglycemic events because the decreased food intake by the mother and glucose transfer to the fetus contribute to hypoglycemia.

What is the correct terminology for an abortion in which the fetus dies but is retained within the uterus? a. Inevitable abortion b. Missed abortion c. Incomplete abortion d. Threatened abortion

b. Missed abortion Missed abortion refers to the retention of a dead fetus in the uterus. An inevitable abortion means that the cervix is dilating with the contractions. An incomplete abortion means that not all of the products of conception were expelled. With a threatened abortion, the woman has cramping and bleeding but no cervical dilation.

A new mother with a thyroid disorder has come for a lactation follow-up appointment. Which thyroid disorder is a contraindication for breastfeeding? a. Hyperthyroidism b. Phenylketonuria (PKU) c. Hypothyroidism d. Thyroid storm

b. Phenylketonuria (PKU) PKU is a cause of mental retardation in infants; mothers with PKU pass on phenylalanine and therefore should elect not to breastfeed. A woman with either hyperthyroidism or hypothyroidism would have no reason not to breastfeed. A thyroid storm is a complication of hyperthyroidism and is not a contraindication to breastfeeding.

Which laboratory marker is indicative of disseminated intravascular coagulation (DIC)? a. Bleeding time of 10 minutes b. Presence of fibrin split products c. Thrombocytopenia d. Hypofibrinogenemia

b. Presence of fibrin split products Degradation of fibrin leads to the accumulation of multiple fibrin clots throughout the body's vasculature. Bleeding time in DIC is normal. Low platelets may occur but are not indicative of DIC because they may be the result from other coagulopathies. Hypofibrinogenemia occurs with DIC.

As part of the discharge teaching, the nurse can prepare the mother for her upcoming adjustment to her new role by instructing her regarding self-care activities to help prevent postpartum depression (PPD). Which statement regarding this condition is most helpful for the client? a. Stay home and avoid outside activities to ensure adequate rest. b. Be certain that you are the only caregiver for your baby to facilitate infant attachment. c. Keep your feelings of sadness and adjustment to your new role to yourself. d. Realize that PPD is a common occurrence that affects many women.

d. Realize that PPD is a common occurrence that affects many women. Should the new mother experience symptoms of the baby blues, it is important that she be aware that these symptoms are nothing to be ashamed of. As many as 10% to 15% of new mothers experience similar symptoms. Although obtaining enough rest is important for the mother, she should not distance herself from her family and friends. Her spouse or partner can communicate the best visiting times to enable the new mother to obtain adequate rest. It is also important that she not isolate herself at home by herself during this time of role adjustment. Even if breastfeeding, other family members can participate in the infant's care. If depression occurs, then the symptoms will often interfere with mothering functions; therefore, family support is essential. The new mother should share her feelings with someone else and avoid overcommitting herself or feel as though she has to be superwoman. A telephone call to the hospital "warm line" may provide reassurance with lactation issues and other infant care questions. Should symptoms continue, a referral to a professional therapist may be necessary.

Another common pregnancy-specific condition is pruritic urticarial papules and plaques of pregnancy (PUPPP). A client asks the nurse why she has developed this condition and what can be done. What is the nurse's best response? a. PUPPP is associated with decreased maternal weight gain. b. The rate of hypertension decreases with PUPPP. c. This common pregnancy-specific condition is associated with a poor fetal outcome. d. The goal of therapy is to relieve discomfort.

d. The goal of therapy is to relieve discomfort. PUPPP is associated with increased maternal weight gain, increased rate of twin gestation, and hypertension. It is not, however, associated with poor maternal or fetal outcomes. The goal of therapy is simply to relieve discomfort. Antipruritic topical medications, topical steroids, and antihistamines usually provide relief. PUPPP usually resolves before childbirth or shortly thereafter.

What is the importance of obtaining informed consent when educating a client regarding contraceptive methods? a. Contraception is an invasive procedure that requires hospitalization. b. The method may require a surgical procedure to insert a device. c. The contraception method chosen may be unreliable. d. The method chosen has potentially dangerous side effects.

d. The method chosen has potentially dangerous side effects. Being aware of the potential side effects is important for couples who are making an informed decision about the use of contraceptives. The only contraceptive method that is a surgical procedure and requires hospitalization is sterilization. Some methods have greater efficacy than others, and this efficacy should be included in the teaching.

A woman with preeclampsia has a seizure. What is the nurse's highest priority during a seizure? a. To insert an oral airway b. To suction the mouth to prevent aspiration c. To administer oxygen by mask d. To stay with the client and call for help

d. To stay with the client and call for help If a client becomes eclamptic, then the nurse should stay with the client and call for help. Nursing actions during a convulsion are directed toward ensuring a patent airway and client safety. Insertion of an oral airway during seizure activity is no longer the standard of care. The nurse should attempt to keep the airway patent by turning the client's head to the side to prevent aspiration. Once the seizure has ended, it may be necessary to suction the client's mouth. Oxygen is administered after the convulsion has ended.

A pregnant woman at 33 weeks of gestation is brought to the birthing unit after a minor automobile accident. The client is experiencing no pain and no vaginal bleeding, her vital signs are stable, and the fetal heart rate (FHR) is 132 beats per minute with variability. What is the nurse's highest priority? a. Monitoring the woman for a ruptured spleen b. Obtaining a physician's order to discharge her home c. Monitoring her for 24 hours d. Using continuous electronic fetal monitoring (EFM) for a minimum of 4 hours

d. Using continuous electronic fetal monitoring (EFM) for a minimum of 4 hours Monitoring the external FHR and contractions is recommended after blunt trauma in a viable gestation for a minimum of 4 hours, regardless of injury severity. Fetal monitoring should be initiated as soon as the woman is stable. In this scenario, no clinical findings indicate the possibility of a ruptured spleen. If the maternal and fetal findings are normal, then EFM should continue for a minimum of 4 hours after a minor trauma or a minor automobile accident. Once the monitoring has been completed and the health care provider is reassured of fetal well-being, the client may be discharged home. Monitoring for 24 hours is unnecessary unless the ERM strip is abnormal or nonreassuring.


Conjuntos de estudio relacionados

Chapter 10 - Honors World History Short Answers

View Set

Chapter 3 Quiz: Medical, Legal, and Ethical Issues

View Set

Quiz 2 Music 101 Loyola Chp: 4,5

View Set

Ch. 20 Caring for Clients with Upper Respiratory Disorders

View Set

U.S. History Fall Final Exam Review

View Set